You are on page 1of 184

2009 (

) 31

12020062008

(1)

(2)

(3)

NCTU PRESS

ii


2008/10/27

NCTU PRESS

iii

Chapter 1
Linear equations
and matrix algebra


http://press.nctu.edu.tw

0 0 2
1. Let A = 1 2 6 . Is the matrix A nonsingular? Justify your answer. If A is
3 7 9

nonsingular, find A1 and express A as a product of elementary row matrices.


( 97)

()()

0 0 2
1 2
det( A) = 1 2 6 = 2 = 2 0 A
3 7
3 7 9

Gauss-Jordan A1
0 0 2 1 0 0 1 2 6 0 1 0 1 2 6 0 1 0

A I = 1 2 6 0 1 0  E1 0 0 2 1 0 0 
E2 0 0 2 1 0 0
3 7 9 0 0 1 3 7 9 0 0 1 0 1 9 0 3 1
1 2 6 0 1 0 1 0 24 0 7 2 1 0 24 0 7 2

E3 0 1 9 0 3 1 
 E4 0 1 9 0 3 E5 0 1 9 0 3
1  1
0 0 2 1 0 0 0 0 2 1 0 0 0 0 11 2 0 0
1 0 24 0 7 2 1 0 0 12 7 2

E6 0 1 0 9 2 3
 1 
E7 0 1 0 9 2 3 1
0 0 1 1 2 0 0 0 0 1 1 2 0 0

12 7 2
A = 9 2 3
1
1
1 2 0 0

E7  E1 A I = I E7  E1 = I A1

( )
1 1
A1 = E7  E1 A = A1 = ( E7  E1 ) = E11  E71

Ei Ei1


http://press.nctu.edu.tw

0 1 0 1 0 0 1 0 0 1 2 0
E1 = 1 0 0 , E2 = 0
1 0 , E3 = 0
0 1 , E4 = 0
1 0 ,
0 0 1 3 0 1 0 1 0 0 0 1
1 0 0 1 0 0 1 0 24

E5 = 0 1 0 , E6 = 0
1 9 , E7 = 0
1 0
0 0 1 2 0 0 1 0 0 1

0 1 0 1 0 0 1 0 0 1 2 0
E = 1
1
1
0 0 , E2 = 0
1
1 0 , E3 = 0 0 1 , E4 = 0 1 0 ,
1 1

0 0 1 3 0 1 0 1 0 0 0 1
1 0 0 1 0 0 1 0 24
E5 = 0
1
1 0 , E6 = 0
1
1 9 , E7 = 0 1 0
1

0 0 2 0 0 1 0 0 1

1 0 3 x1 5
2. Consider the linear system Ax = b given by 2 1 5 x2 = 8 . If the
4 1 s x3 t

matrix A is not invertible, what is the value of s? Apply this value of s and find
the value of t that makes the linear system Ax = b have a solution.
( 97)

1 0 3 5 1 0 3 5 1 0 3 5
2 1 5 8 0 1 1
2 0 1 1 2

4 1 s t 0 1 s 12 t 20 0 0 s 11 t 18

A 3 s = 11
t = 18 (zero row)


http://press.nctu.edu.tw

1 2 3 1 b
3. Suppose the matrix 2 5 3 a 0 can be transformed to the reduced row
1 0 8 6 c

1 0 0 2 0
echelon form 0 1 0 d 1 . Which of the following equalities are
0 0 1 1 e

correct?
(a) a = 1.
(b) b = 3.
(c) c = 40 3.
(d) d = 1.
(e) e = 2.
( 97)

1 2 3 1 b 1 2 3 1 b 1 0 0 2 0
2 5 3 a 0 0 1 3 a 2 2b  0 1 0 d 1

1 0 8 6 c 1 0 8 6 c 0 0 1 1 e

1 2 3 1 1 2 3 b
2 5 3 x1 = a 2 5 3 x 2 = 0

1 0 8 6 1 0 8 c

2
x1 = d
1

0 1 2 3 2 1
x 2 = 1 2 5 3 d = a d = 0 a = 1

e 1 0 8 1 6


http://press.nctu.edu.tw

1 2 3 0 b
2 5 3 1 = 0 e = 5 b = 3 c = 40
3 3
1 0 8 e c

(b)(c)

0 1 2
4. Find the PLDU factorization of a matrix A = 0 6 3 , and use the PLDU
7 5 4

6
factors of A to solve Ax = 8 .
3

( 95)

0 1 2 7 5 4 7 5 4 1 5 7 4 7

A = 0 6 3  1 S (1 7,1 6, 2 3) 0 1 1 2 = U
P31 0 6 3 E32 ( ) 0 6 3 
6

7 5 4 0 1 2 0 0 3 2 0 0 1

SE32 P31 A = U

0 0 1 1 0 0 1 7 0 0

P31 = 0 1 0 E32 = 0 1 0 S = 0 1 6 0

1 0 0 0 1 6 1 0 0 2 3

A PLDU

A = P311 E321 S 1U = PLDU

0 0 1 1 0 0 7 0 0 1 5 7 4 7

P = 0 1 0 L = 0 1 0 D = 0 6 0 U = 0 1 1 2
1 0 0 0 1 6 1 0 0 3 2 0 0 1

PLDU PLDU


http://press.nctu.edu.tw

A1 = U 1 D 1L1 P 1

6 6

1 1 1 1 1
x = A 8 = U D L P 8
3 3

6 3 3 37
x = U SE32 P31 8 = U SE32 8 = U S 8 = U 4 3
1 1 1 1

3 6 14 3 28 9

1 5 7 4 7 37
0 1 1 2 x = 4 3 U

0 0 1 28 9


25 2 28
x1 = x2 = x3 =
21 9 9

1 2 3 1 0 0 u11 u12 u13


5. Let 2 4 7 = l21 1 0 0 u22 u23 . Which of the followings are
3 5 3 l31 l32 1 0 0 u33

correct?
(a) u11 = 1
(b) u12 = 2
(c) l21 = 2
(d) It is impossible to have the above equation, so no way to have u11 , u12 ,
etc.
(e) u33 = 0
( 95)

1 2 3 3 5 3 3 5 3 3 5 3
2 4 7 1, 3 2 4 7 0 2 3 5 0 2 3 5 = U

3 5 3 1 2 3 0 1 3 2 0 0 1 2


http://press.nctu.edu.tw

1 0 0

L = 2 3 1 0
1 3 1 2 1

0 0 1 1 0 0 3 5 3

PA = 0 1 0 A = LU = 2 3
1 0 0 2 3 5
1 0 0 1 3 1 2 1 0 0 1 2

LU
(d)

6. Please determine the lower triangular matrix L which can decompose the
following symmetric matrix B according to B = LLT .
a c d 0 d f
B= = LLT =
c b f e 0 e
( 97)

T = LD
LU B B = LDL 1 2 D1 2 LT = ( LD
1 2 )( LD
1 2 )T

D
1 0 a 0 c
a c 2
1
B= = c c a = LDL
T
c b 1 0 b
a a 0 1
c
a a 0
a
= c 2 = LLT
c 2
b c
a a 0 b
a
a 0 ba c 2

http://ccjou.twbbs.org/blog/?p=4237


http://press.nctu.edu.tw

1 5 1 2
7. Given 3 3 matrix A and four vectors a = 1 , b = 3 , c = 3 , d = 2

0 2 1 3

satisfying Aa = b, Ab = c, Ac = d, find Ad.


( 96)

1 5 1 5 1 2
A 1 3 3 = 3 3 2
0 2 1 2 1 3

1
5 1 2 1 5 1
A = 3 3 2 1 3 3 A
2 1 3 0 2 1

d
Ad

5 1 2 1 5 1 2
1
1 5 1 2

Ad = 3 3 2 1 3 3 2 1 3 3 x = 2

2 1 3 0 2 1 3 0 2 1 3

5 1 2 2
Ad = 3 3 2 x x = 1

2 1 3 1

5 1 2 2 7
Ad = 3 3 2 1 = 5
2 1 3 1 2


http://press.nctu.edu.tw

8. What is the solution of


1 2 3 1 1 2 3 4 1 2 3 1 1 2 3 4
2 2 1 2 5 6 7 8 2 2 1 2 5 6 7 8
+ ?
3 1 1 3 0 0 0 0 3 1 1 3 0 0 0 0

1 2 3 2 9 10 11 12 1 2 3 2 9 10 11 12
2 4 6 8
20 24 28 32
(a)
64 72 80 88

0 0 0 0
2 4 6 8
20 24 28 32
(b)
64 72 88 80

0 0 0 0
2 4 6 8
20 24 28 32
(c)
64 72 88 80

0 0 4 0
2 4 6 8
20 24 28 32
(d)
64 76 88 80

0 0 0 0
2 4 6 8
20 24 28 32
(e)
60 76 88 88

0 0 4 0
( 95)

AB + CB ( A + C ) B

2 0 0
0 1 2 3 4
0 4 2
0 5 6 7 8

0 2 2
6 0 0 0 0

0 0 6
0 9 10 11 12
5 34


http://press.nctu.edu.tw

1 2 3 4
5 6 7 8
=
0 2 2 6 0 0 0 0 64 72 80 88

0 0 6 0 9 10 11 12 0 0 0 0
(a)

9. The commutator of two n n matrices A and B is defined as [ A, B ] = AB BA.


Let 0 denote the n n zero matrix. For n n matrices A, B, and C, which of
the following statements is NOT correct?
(a) [ A, B ] = [ B, A].
(b) [ A, B + C ] = [ A, B ] + [ A, C ].
(c) [ A, BC ] = [ A, B ]C + B[ A, C ].

(d) [ A,[ B, C ]] + [ B,[ A, C ]] + [C ,[ A, B]] = 0.


(e) If [ A, B ] = 0 and [ B, C ] = 0, then [ A, C ] = 0.
( 97)

(a) [ B, A] = ( BA AB ) = AB BA = [ A, B]

(b) [ A, B + C ] = A( B + C ) ( B + C ) A = ( AB BA) + ( AC CA) = [ A, B ] + [ A, C ]


(c) [ A, BC ] = ABC BCA = ABC BAC + BAC BCA
= ( AB BA)C + B ( AC CA) = [ A, B ]C + B[ A, C ]

(d) [ A,[ B, C ]] + [ B,[ A, C ]] + [C,[ A, B]] 12 6 6


ABC 6

[ A,[ B, C ]] = [ A, BC CB ] = ABC ACB ( BCA CBA)


[ B,[ A, C ]] = [ B, AC CA] = BAC BCA ( ACB CAB)
[C ,[ A, B]] = [C , AB BA] = CAB CBA ( ABC BAC )

[ A,[ B, C ]] + [ B,[ A, C ]] + [C,[ A, B]] = 2 [ B,[ A, C ]] 2 [[ A, C ], B] 0

[ B,[ A, C ]]

[ B,[C , A]]


http://press.nctu.edu.tw

(e) [ A, B ] = 0 = AB BA [ B, C ] = 0 = BC CB A B
B C A C
B = I [ A, B ] = [ A, I ] = AI IA = A A = 0 [ B, C ] = [ I , C ] = 0
[ A, C ] = 0
(d)(e)

http://ccjou.twbbs.org/blog/?p=4478

10. Which of the followings are correct?


(a) If A, B R mk , then calculating A + B costs O ( mk ).
(b) If A R mk , B R k n , then calculating AB costs O ( nmk ).
(c) If A R mm is invertible, using Gaussian elimination to find A1 costs
O(m3 ).
(d) If A R mk , B R k m , u R m1 and k  m, then calculating ( AB )u
costs less than A( Bu).
(e) If A R mk , B R k m , u R m1 and k  m, then calculating ( AB )u
needs more storage than A( Bu).
( 95)

(a) A + B mk O ( mk )
(b) AB mn k ( k 1)
O ( nmk )
(c) A

m(m 1) + (m 1) ( m 2 ) +  + 1
m m m
1 1 1
i(i 1) = i i = 6 m ( m + 1) (2m + 1) 2 m(m + 1) 3 m
i =1 i =1
2

i =1
3

m(m 1) 1 2
(m 1) + (m 2) +  + 1 = m
2 2
O ( m3 )
(d) (b) ( AB )u O (km 2 + m 2 ) = O (km 2 ) A( Bu)
O ( km + km) = O (km) ( AB )u A( Bu )
(e)

( AB )u A R m k
B R k mu R m1 AB R mm
mk + mk + m + m 2 = m 2 + 2mk + m k  m O (m 2 ) A( Bu )
mk + mk + m + k = 2km + m + k O ( km )


http://press.nctu.edu.tw

( AB )u A( Bu)
(a)(b)(c)(e)

( A + UV )
1
11. Let A R nn , U , V R nk . What is T
? Assume A and

(I +V T
A1U ) are invertible.

( )
1
(a) A1U I + V T A1U V T A 1

( )
1
(b) A1 A1U I + V T A1U V T A1

( )
1
(c) A1 A1 I + V T A1U V T A 1

A1 A1U ( I + V T A1U ) UV T A1
1
(d)

A1 A1UV T ( I + V T A1U ) V T A1
1
(e)

( 95)

B = A + UV T
A A1

( )
BA1 = A + UV T A1 = I + UV T A1

BA1U = U + UV T A1U = U I + V T A1U ( )


(I +V ) ( )
1
T
A1U I + V T A1U

BA1U ( I + V T A1U ) = U U
1


BA1 = I + UV T A1 UV T A1 = BA1 I V T A1

BA1U ( I + V T A1U ) V T A1 = UV T A1 = BA1 I


1


http://press.nctu.edu.tw

BA1 BA1U ( I + V T A1U ) V T A1 = I B


1

B 1 = A1 A1U ( I + V T A1U ) V T A1
1

(b)

http://ccjou.twbbs.org/blog/?p=1320

1 0 0 0 1 0 0 0
2 3 0 0 0 1 0 0
12. Let matrices A = , I = and
0 4 5 0 0 0 1 0

0 0 6 7 0 0 0 1
1
B = ( I + A) ( I A), calculate the matrix ( I + B ) 1.
( 96)

A () 0

B A
4 4
B ( I + A) ( I + A) 1
( I + A) B = B + AB = I A AB + A + B = I
( I + A)( I + B ) = 2 I
( I + B ) 1 ( I + B ) 1
1
( I + B ) 1 = ( I + A)
2
1 0 0 0
1 2 0 0
( I + B ) 1 =
0 2 3 0

0 0 3 4

http://ccjou.twbbs.org/blog/?p=1086


http://press.nctu.edu.tw

1
13. If A R nn and A2 A + I n = 0, then ( A + 2I n ) =?

( 98)

( A + 2 I )1
A I A + 2 I
A2 A + I = ( A + 2 I )( A 3I ) + 7 I = 0
1
( A + 2 I )( A 3I ) = 7 I ( A + 2 I )1 = ( A 3I )
7

1
1

14. If u = 1 then ( I 5 + 2uuT )( I 5 + uuT ) 1 u = ?

1
1
( 97)

5 5 ( I 5 + uuT ) 1

( I 5 + uuT ) 1 ( I 5 + uuT ) 1

x = ( I 5 + uuT ) 1 u

( I 5 + uuT ) 1 ( I 5 + uuT )

( I 5 + uuT )x = x + u (uT x) = u( uT x )

2
x u x = ku k + k u = 1

2 1
u = uT u = 5 k = 1 6 x = u
6


http://press.nctu.edu.tw

1 1 11
6
( 6
)
( I 5 + 2uuT ) u = u + 2u(uT u) = u
6

http://ccjou.twbbs.org/blog/?p=1423


http://press.nctu.edu.tw

Chapter 2
Vector spaces


http://press.nctu.edu.tw

0 1 1 4
3
1 2 1 1 2
1. For the linear system Ax = b, where A = , answer the
1 2 5 13 5

1 3 0 2 2
following questions.
(a) Find the rank of A and a basis for the column space of A.
(b) Find a basis for the nullspace N ( A). What is the dimension of N ( A) ?
( 96)

(reduced row echelon form)


(rank)(nullspace)

0 1 1 3 4 1 2 1 1 2 1 2 1 1 2
1 2 1 1 2 0 1 1 3 4 0 1 1 3 4

1 2 5 13 5 1 2 5 13 5 0 4 4 12 3

1 3 0 2 2 1 3 0 2 2 0 1 1 3 0
1 2 1 1 2 1 0 3 7 0
0 1 1 3 4 0 1 1 3 0

0 0 0 0 1 0 0 0 0 1

0 0 0 0 0 0 0 0 0 0

(a) 3 (pivot) rank( A) = 3 125


A () 125

0 1 4

1 2 2
, ,
1 2 5
1 3 2

(b) (nullspace matrix)

3 7
1 3

N = 1 0 N ( A) N

0 1
0 0


http://press.nctu.edu.tw

3 7

1 3

1 , 0 dim N ( A) = 2
0 1

0 0

http://ccjou.twbbs.org/blog/?p=1367

2. Use Gaussian elimination procedure to find the reduced row echelon form, rank,
and nullity of the matrix below:
1 0 2 1 0 1
2 1 6 2 0 4

0 1 2 1 1 1

1 2 6 3 1 2
( 96)


1 0 2 0 0 3
0 1 2 0 0 2

0 0 0 1 0 2

0 0 0 0 1 1
4 (rank) 4 6
6 4 = 2 (nullity) 2

1 2 3 4
8 7 6 5
3. If P = , then rank( P) = ?
9 10 11 12

16 15 14 13
( 98)

P
rank(P)
3


http://press.nctu.edu.tw

1 4 2

1 2 3 4 1 2 3 4 1 2 3 4
8 7 6 5 8 7 6 5
P= 8 7 6 5
9 10 11 12 8 8 8 8 1 1 1 1

16 15 14 13 8 8 8 8 1 1 1 1
2 1
1 2 3 4 1 2 3 4
9 9 9 9
1 1 1 1
1 1 1 1 0 0 0 0

1 1 1 1 0 0 0 0
P 2 rank(P) = 2

4. We decide to use a matrix storing all web connections. If web i has n out-links
and j is one site that it connects to, then we put the ij element to be 1/n.
Otherwise the ij element is zero. If n = 0, then the ij element is zero. Which of
the followings are incorrect?
(a) Zero rows are possible as some pages have no out-link.
(b) Zero columns are possible as some pages are never linked.
(c) The rank of this matrix > (# total web pages 1 )
(d) Sum of each row is 0 or 1.
(e) Sum of each column is 1.
( 95)

i j

(a) n = 0
(b)
(c) W m m
rank(W ) m rank(W ) = m > ( m 1)

0 1 2 1 2
W = 1 0 0 rank(W ) (m 1)
0 1 0


http://press.nctu.edu.tw

0 1 2 1 2
W = 1 0 0
0 0 0

(d) n > 0 1 n = 0 0
(e) (c) 1
(c)(e)

5. If A R 87 and rank( A) = 3, then nullity( AT ) = ?


( 97)

nullity( AT ) = dim N ( AT )
(rank-nullity) A m n
rank( A) + dim N ( AT ) = m rank( A) + dim N ( A) = n
nullity( AT ) = m rank( A) = 8 3 = 5

m n m n
rank( A)
dim C ( A) = rank( A)
nullity( AT ) = dim N ( AT ) = m rank( A)
dim C ( AT ) = rank( AT ) = rank( A)
nullity( A) = dim N ( A) = n rank( A)

http://ccjou.twbbs.org/blog/?p=1327


http://press.nctu.edu.tw

6. Which of the following statements are correct?


(a) For an n n matrix A, the columns of A are linearly independent if and
only if the rows of A are linear independent.
(b) For an m n matrix A, the nullity of A equals the nullity of its transpose
AT .

(c) An m n matrix A defines some linear transformation TA : R n R m . TA

is onto if and only if rank A = m.


(d) A set S of vectors forms a basis for a subspace V of R n if and only if the
vectors of S are linear independent and the number of vectors in S equals the
dimension of V.

x1

(e) The set V = x2 R : 3x1 + 2 x2 x3 = 1 is not a subspace of R 3 .
3

x
3
( 97)

(a) n n (equivalent)

(full rank) rank( A) = n dim C ( A ) = dim C ( AT ) = n


rank( A) < n m n

1 1
m n A = 0 1
0 1

(b)

(b) dim N ( A ) = n rank ( A )

( ) ( )
dim N AT = m rank( A) dim N ( A) dim N AT = n m nm

dim N ( A) = dim N ( AT )
(c)

TA (onto) Range ( A ) = R m rank( A) = Range( A) = m

( )
dim N AT = m dim Range( A) = 0


http://press.nctu.edu.tw

(d)

0
(e) 0 V V
0

(a)(c)(d)(e)

7. If the rank of the set of vectors b1 = (0,1, 1), b 2 = ( a, 2,1), b 3 = (b,1, 0) is equal
to the rank of the set of vectors a1 = (1, 2, 3), a 2 = (3, 0,1), a3 = (9, 6, 7) and
b 3 can be represented as the linear combination of a1 , a 2 , a3 , find the values of a
and b.
( 96)

b 3 a1 a 2 a3

x1
[a1 a 2 a3 ] x2 = b3
x3


1 3 9 b 1 3 9 b 1 3 9 b
2 0 6 1 0 6 12 1 2b 0 6 12

1 2b
3 1 7 0 0 10 20 3b 10 5
0 0 0 3b b +
3 3
10 5
3b b + = 0 b = 5
3 3

rank ([a1 a 2 a3 ]) = 2 rank ([b1 b 3 b 2 ]) = 2 b 2

b1 b 3

0 5 a 1 1 2 1 1 2 1 1 2
1 1 2 0 5 a 0 5 a 0 5
a

1 0 1 1 0 1 0 1 3 a
0 0 3
5
a = 15


http://press.nctu.edu.tw

8. Consider the vectors u1 = ( 0,1, 0,1,0 ) , u 2 = (1, 0, 0,0, 0 ) , u3 = (1, 0,1, 0,1) ,

w1 = (1,1, 0,0, 0 ) , w 2 = (1, 2, 0,1,0 ) , w 3 = (1,1,1, 0,1) . Let U be the subspace of

R 5 spanned by u1 , u 2 and u3 , and W be the subspace spanned by w1 , w 2

and w 3 .
(a) Select bases for U and W from u1 , u 2 and u 3 , and w1 , w 2 and w 3 ,
respectively.
(b) What are the dimensions of U and W?
(c) Determine U W .
(d) Find a basis of U W .
(e) Extend the basis from (d) to bases of U and W in such a way that you will

get a basis of U + W = span (U W ) as well. What is the dimension of

U +W ?
( 97)

(a) {u1 , u 2 , u3 }

{w1 , w 2 , w 3 } (span) U
W
(b) (a) dim U = 3 dim W = 3
(c) U W

u1T 0 1 0 1 0 1 0 0 0 0
T
u 2 = 1 0 0 0 0 0 1 0 1 0
uT3 1 0 1 0 1 0 0 1 0 1

w1T 1 1 0 0 0 1 0 0 1 0
T
w 2 = 1 2 0 1 0 0 1 0 1 0
w T3 1 1 1 0 1 0 0 1 0 1


http://press.nctu.edu.tw

0 0

1 0

U W = span 0 , 1
1 0

0 1

0 0

1 0
(d) U W 0 , 1 (c)span
1 0

0 1

(e) (c) 4 dim (U W ) = 4

0 0 1 1

1 0 0 0
0 , 1 , 0 , 0
1 0 0 1

0 1 0 0

http://ccjou.twbbs.org/blog/?p=3456

9. Assume A is an m n matrix with rank r and b is a column vector. Which


statements are true?
(a) If m > r and n = r, then Ax = b must have no solution for some b and
exactly one solution for other b.
(b) If m > r and n > r, then Ax = b has infinitely many solutions for some b and
exactly one solution for other b.
(c) If n = r, then Ax = b has either one solution or none.
( 98)

(a) m > r A b A
Ax = b n = r A


http://press.nctu.edu.tw

(b) n > r A

(c) (a)

http://ccjou.twbbs.org/blog/?p=1501

1 1 1
1 1 1
10. Suppose Q = [q1 q 2 q3 ] = . Let S = span(q , q ) and
1 1 1 12 1 2


1 1 1
S 23 = span(q 2 , q3 ). Which statements are true?
(a) The union of the two subspaces S12 and S 23 forms a vector space.
(b) The intersection of the two subspaces S12 and S 23 forms a vector space.
(c) The span(q1 ) is an orthogonal complement of the subspace S 23 .
(d) The rows of Q forms a basis for the row space.
(e) The dimension of the row space of Q is 3.
( 98)

(a) vector space subspace


(b) vector space subspace

(c) Q R 4 rank S 23 = 2 span(q1 )

1 an the
(d) Q
(e) dim C (QT ) = 3
(b)(e)

11. If A M mn ( R), prove that N ( AT A ) = N ( A) and deduce that

rank ( AT A ) = rank A.

( 96)

N ( AT A ) = N ( A) N ( A) N ( AT A )


http://press.nctu.edu.tw

( )
N AT A N ( A)

N ( A) N ( AT A ) x N ( A) Ax = 0

AT AT Ax = AT 0 = 0
x N ( AT A)

( )
N AT A N ( A) x N AT A ( )
AT Ax = 0

xT xT AT Ax = ( Ax ) ( Ax ) =
T 2
Ax = 0 Ax = 0 x N ( A)

A m n rank( A) + dim N ( A) = n AT A

n n rank ( AT A ) + dim N ( AT A ) = n

( ) ( )
N AT A = N ( A) rank AT A = rank (A)

http://ccjou.twbbs.org/blog/?p=3984


http://press.nctu.edu.tw

12. Prove or disprove the following statement: In a vector space V, if vi and v j

are linear independent for i, j = 1,2,3, i j, then v1 , v 2 , v 3 are linear

independent.
( 97)

1 0 1
V = R 2 v1 = v 2 = v3 =
0 1 1
v1 v 2 v3

v v


v v

v

http://ccjou.twbbs.org/blog/?p=1431
http://ccjou.twbbs.org/blog/?p=4240


http://press.nctu.edu.tw

13. Let F ( R ) denote the set of all functions from R to R. Choose the following
subsets of F ( R ) which are linear independent.

(a) {t 2
2t + 5, 2t 2 4t + 10}

(b) {sin t, sin 2 t , cos 2 t , 1}

(c) {t 2
2t + 5, 2t 2 5t + 10, t 2 }

(d) {t , t sin t}

(e) {e , e
t 2t
, , ent ,}

( 97)


c1v1 +  + cn v n = 0
c1 =  = cn = 0 v1 , , v n

(a) (2) t 2 2t + 5 + [ 2t 4t + 10] = 0

(b) 0 [sin t ] + sin 2 t + cos 2 t (1) [1] = 0

1 2 1
(c) 2 5 0
5 10 0

1 2 1
0 1 2

0 0 5


(d)
(e) (d)
(c)(d)(e)

http://ccjou.twbbs.org/blog/?p=1388


http://press.nctu.edu.tw

14. Let V and W be vector spaces over a field F of dimensions m and n, respectively.

Let Z be a vector space given by Z = {( v, w ) : v V and w W }. What is the

dimension of Z?
( 96)

Z = {( v, w ) : v V and w W } (Cartesian product)Z

Z = V W direct sum

( v1 , w1 ) + ( v 2 , w 2 ) = ( v1 + v 2 , w1 + w 2 )
c( v1 , w1 ) = (cv1 , cw1 )

V {0} Z = V W V {0} V (isomorphic)


(one-to-one)(onto){0} W
W Z x (V ) y
x x 0
(W ) direct sum = + x y {0} (
y 0 y
Z ) Z = V W
V W
dim V W = dim V + dim W

V W {v1 , , v r } {w1 , , w s }

{v1 , , v r , w1 , , w s } Z {v1 , , v r } {w1 , , w s }


Z
dim Z = r + s = dim V + dim W

15. Let T be a linear operator from Cn to itself and C. Let N(T) denote the
null space of T. Prove that for every basis of Cn with respect to which T has an
upper-triangular matrix, appears on the diagonal of the matrix of T precisely
dim N ((T I ) n ) times, or disprove it by giving a counter example.
( 98)


http://press.nctu.edu.tw

rank(T I )n = r
dim N ((T I )n ) = n rank(T I )n = n r
T I (n r ) T I r
rank(T I )n = r

T I
Schur
A P = U AU U
(unitary matrix) U = U 1 Jordan form J = M 1 AM
B C 1
J Jordan T I T I = M M
0 D
B (n r ) (n r ) B D
r r D

Bn E 1
(T I )n = M n
M B n = 0 D n
0 D

( )
n
D n = [ D ]ii 0 M
ii

Bn E E
rank (T I )
n
= rank n
= rank n = r
0 D D

T I = D + U D U
(T I ) n = D n + S + U n
S + U n
rank(T I ) n = rank D n = rank D = rank(T I ) = r

http://ccjou.twbbs.org/blog/?p=4273


http://press.nctu.edu.tw

Chapter 3
Linear transformations


http://press.nctu.edu.tw

1. Let T : R 3 R 3 be the linear transformation defined by

T (x) = ( x1 x3 , 2 x1 + 3 x2 x3 ,3 x1 3 x2 )T .

(a) Find the standard matrix representation A for the linear operator T.
(b) Find the LU decomposition of A, where L is a unit lower triangular matrix
and U is an upper triangular matrix.
(c) Find a basis for the column space of A.
(d) Find a basis for the nullspace of A.
( 96)

x1 1 0 1
(a) A T (x) = Ax = A x2 A = 2 3 1

x3 3 3 0

(b) L

1 0 1 1 0 0 1 0 1
A = 2 3 1 = 2 1 0 0 3 3 = LU
3 3 0 3 1 1 0 0 0

(c) U 12 A

1 0

2 , 3
3 3

1 0 1 1 0 1
(d) U U = 0 3 3 0 1 1
0 0 0 0 0 0

1

1
1


http://press.nctu.edu.tw

2. Define a linear transformation T from R 3 to R 3 as follows:

1 1 0 1 1 1

T 0 = 0 , T 1 = 0 , T 0 = 2 .
1 1 0 1 1 1

(a) Find the matrix of the linear transformation T.
(b) Find a basis of the kernel of the transformation T.

2

(c) Find the coordinate vector of T 1 with respect to the basis
2

1 1 1

1 , 1 , 0 .
1 0 0

( 97)


(a) T

1 0 0

0 , 1 , 0
0 0 1

T ( v ) = Av A 3 3 v R 3

1 0 1 1 1 1
A 0 1 0 = 0 0 2
1 0 1 1 1 1

1
1 1 1 1 0 1 0 1 1
A = 0 0 2 0 1 0 = 1 0
1
1 1 1 1 0 1 1 1 0


http://press.nctu.edu.tw

1 1 1 1 1 0
1 1 1 1
T 0 = T 0 + 0 = T 0 + T 0 = 1
0 2 1 2 1 2 1 2 1 1

0 1

T 1 = 0
0 1

0 1 1 1 1 1
1 1 1 1
T 0 = T 0 0 = T 0 T 0 = 1
1 2 1 2 1 2 1 2 1 0

A
(b) ker(T ) = N ( A) N ( A)

0 1 1 1 1 0 1 1 0 1 0 1
A = 1 0 1 1 0 1 0 1 1 0 1 1
1 1 0 0 1 1 0 1 1 0 0 0

1 1

N = 1 N ( A) 1
1 1

2 2 0 1 1 2 1

(c) T 1 T 1 = 1 0 1 1 = 0
2 2 1 1 0 2 1

1 1 1 1 1 1 1 x1 x1 1
0 = x 1 + x 1 + x 0 = 1 1 0 x x = 1
1 2 3 2 2
1 1 0 0 1 0 0 x3 x3 1

2 1

T 1 = 1
2
B 1

http://ccjou.twbbs.org/blog/?p=3864


http://press.nctu.edu.tw

3. For the linear transformation T : R 3 R 2 with T (a1 , a2 , a3 ) = (a1 2a2 ,3a3 ),

find a basis for the nullspace of T, and compute the nullity and rank of T.
( 97)

T (a1 , a2 , a3 ) = (a1 2a2 ,3a3 )

a1 a1
1 2 0
T ( a1 , a2 , a3 ) = A a2 = a2 A
0 0 3
a3 a3

2
1 2 0 1 2 0

0 0 3 0 0 1 N = 1
0

2

1 (nullity) nullity(T ) = dim N ( A) = 1 A
0

2 rank(T ) = 2

Consider the basis S = { v1 , v 2 , v 3 } for R 3 , where v1 = [1, 2,1] , v 2 = [ 2, 9, 0] ,


T T
4.

and v 3 = [3,3, 4] , and let T : R 3 R 2 be the linear transformation such that


T

T ( v1 ) = [1, 0] , T ( v 2 ) = [ 1,1] , and T ( v 3 ) = [ 0,1] . Let w = [ 7,13, 7 ] , find


T T T T

T ( w ).
( 95)

w S w = c1 v1 + c2 v 2 + c3 v 3

1 2 3 c1 7 c1 1
2 9 3 c = 13 c = 1
2 2
1 0 4 c3 7 c3 2

T ( c1v1 + c2 v 2 + c3 v3 ) = c1T ( v1 ) + c2T ( v 2 ) + c3T ( v3 )


http://press.nctu.edu.tw

1 1 0 2
T (w ) = (1) + + 2 =
0 1 1 3

c1
T (w ) = [T ( v1 ) T ( v 2 ) T ( v 3 )] c2 = [T ( v1 ) T ( v 2 ) T ( v 3 )][ w ]S
c3

c1
[ w ]S w S c2
c3

5. Let L be the linear transformation from R 3 to R 3 having the following matrix

cos 0 sin
representation with respect to the standard basis: 0 1 0 .
sin 0 cos

x
(
(a) Find L L ( L ( v ) ) ) , where v = y .
z

(b) Find the dimension of the kernel space.


(c) Find a basis of the range space of L.
( 96)

cos 0 sin x
(a) L( v ) = Av = 0 1 0 y y
sin 0 cos z


3
cos 0 sin x
(
L L ( L ( v )) ) = A v =
3
0 1 0 y

sin 0 cos z

3 3


http://press.nctu.edu.tw

cos 3 0 sin 3 x x cos 3 + z sin 3


(
L L ( L ( v )) ) = 0 1 0 y = y

sin 3 0 cos 3 z x sin 3 + z cos 3

(b) L (
det A = 1 ) nullity( L) = dim N ( A) = 0
(c) L (onto) R ( L ) = C ( A) = R 3 3

1 0 0

R ( L) 0 , 1 , 0
0 0 1

6. Let T : P2 (R ) P3 (R ) be given by T ( f ( x) ) = xf ( x) + f ( x).

(a) What is the nullspace N (T ) of T?


(b) What is the range R(T ) of T?
(c) Is T one-to-one? Is T onto?
( 96)

(a) T ( f ( x) ) = xf ( x) + f ( x). f ( x) = ax 2 + bx + c

( )
T ( f ( x) ) = xf ( x) + f ( x) = x ax 2 + bx + c + 2ax + b
= ax + bx + ( 2a + c ) x + b
3 2

Pk (R ) (ordered basis) {x k , x k 1 , , x,1}

1 0 0
a a a
0 1 0
T T b = A b = b A
2 0 1

c c c
B
0 1 0
1 0 0 1 0 0
0 1 0 0 1 0
A =
2 0 1 0 0 1

0 1 0 0 0 0


http://press.nctu.edu.tw

0
0 N (T ) = {0}
0

(b) A R(T )

1 0 0

0 1 0
{ x + 2 x, x + 1, x}
3 2
, ,
2 0 1
0 1 0

(c) T ( f ( x) ) = T ( g ( x) ) T ( f ( x) ) T ( g ( x) ) = T ( f ( x) g ( x) ) = 0

N (T ) = {0} f ( x) = g ( x) T (one-to-one)

dim R (T ) = 3 dim P3 (R) = 4 R (T ) P3 (R ) T (onto)


f ( x) = ax 2 + bx + c c

7. An affine transformation of R 2 is a function T : R 2 R 2 of the form

T ( x ) = Ax + b, where A is an invertible 2 2 matrix and b R 2 . Which of

the following statements are correct?

(a) T 1 ( x ) = A1x A1b.

(b)Affine transformations map straight lines to straight lines.


(c)There is no affine transformation that can map a straight line to a circle.
(d)Affine transformations map parallel straight lines to parallel straight lines.
(e)There exists an affine transformation that maps parallel straight lines to
intersecting straight lines.
( 97)

(affine transformation)

(a) T 1 (T ( x ) ) = A1T ( x ) A1b = A1 ( Ax + b ) A1b = x

(b) R 2 x(t ) = ta + c a

T ( ta + c ) = A ( ta + c ) + b = t ( Aa ) + ( Ac + b )


http://press.nctu.edu.tw

T
(c) (b)
(d) (b) a Aa

(e) (d)
(a)(b)(c)(d)

8. Given 2D vectors e1 and e 2 , find a Householder matrix H so that H (e1 ) = e 2 .


( 98)

Householder H = I 2uuT u = 1

He1 = ( I 2uuT )e1 = e1 2(uT e1 )u = e 2 u (e1 e 2 )

e1 e 2 1 0
u u = e1 = e 2 =
e1 e 2 0 1

1 1
u=
2 1
1 0 1 1 1 0 1
H = 2 [1 1] =
0 1 2 1 2 1 0

Householder
Householder H H 2 = I

He1 = e 2 H H 2e1 = Ie1 = e1 = He 2

1 0 0 1
H [e1 e 2 ] = H = [e 2 e1 ] = H
0 1 1 0

http://ccjou.twbbs.org/blog/?p=3272


http://press.nctu.edu.tw

9. If u R n and ut u = [3], then there exist k R such that

( I n + uut )10 = I n + kuut , where k = ?

( 98)

(I + uu t ) (I + uu t )
2 3
n n

I n + uut

(I n + uu t ) u = I nu + u (u t u) = 4u I n + uut u 4

(I + uu t ) u = 410 u
10
n

410 1
( )
I n + kuu t u = I nu + ku (ut u) = (1 + 3k )u 410 = 1 + 3k k =
3

10. The field Z 2 consists of two elements 0 and 1 with the operations of addition
(+) and multiplication ( i ) defined by 0+0=0, 0+1=1, 1+0=1, 1+1=0, 0 i 0=0,
0 i 1=0, 1 i 0=0, and 1 i 1=1.

1 1 0 0 x1 0
0 x 0
1 1 1 2
Let A = , x= , and b = , where all entries are in Z 2 .
0 0 1 0 x3 1

1 0 1 1 x4 0

(a) For the matrix A, compute the rank and the inverse if it exists.
(b) Determine whether the system Ax = b is consistent. If the system is
consistent, find all solutions.
(c) Find a basis for the solution set of the corresponding homogeneous system.
( A98)

(a) A


http://press.nctu.edu.tw

1 1 0 0 1 1 0 0 1 1 0 0
0 1 1
1 0 1 1
1 0 1 1 1
A= 4 r1 + r 4
r 4 r2 + r4
r
0 0 1 0 0 0 1 0 0 0 1 0

1 0 1 1 0 1 1 1 0 0 0 0
1 1 0 0 1 0 0 1
0 1 0 1 0 1 0 1
2 r 2 + r3
r 1 r1 + r 2
r =R
0 0 1 0 0 0 1 0

0 0 0 0 0 0 0 0
rank A = 3 A
0
0
(b) Ax = b b = (a)
1

0

0 0 0 0 1
0 0 0 1 1
b= 4 r1 + r 4
r 4 r2 + r4
r 2 r 2 + r3
r 1 r1 + r 2 = c
r
1 1 1 1 1

0 0 0 0 0

1
1
Rx = c x p = R
1

0
1 1
1 1
N = Ax = 0 x h =
0 0

1 1
1 1 1 0
1 1 1 0
x p = x = x p + xh = + =
1 1 0 1

0 0 1 1
1
1
(c) (b) N
0

1


http://press.nctu.edu.tw

p ( 1)

11. A mapping T : P R2 3
defined by T ( p ) = p ( 0 ) is a linear
p (1)

transformation. Here the vector space P 2 is the set of all polynomials of degree
2 or less; and R 3 denotes the collection of all lists of three real numbers,
usually written as 3 1 column matrices. Find the matrix for T relative to the

basis {1, x, x 2 } for P 2 and the standard basis for R 3 .

( 95)

a
p ( x) = a + bx + cx [ p ]B
2
= b
c

p ( 1)

p (1) = a b + c p (0) = a p (1) = a + b + c T ( p ) = p ( 0 )
p (1)

a a b + c
[T ( p ) ]B = A b = a
c a + b + c

1 1 1
A = 1 0 0
1 1 1

A = T (1) T ( x) T ( x 2 ) A

(image)

p ( 1)

T ( p ) = p ( 0 ) p ( x) = 1 p ( x) = x p ( x) = x 2
p (1)

1 1 1
T (1) = 1 T ( x ) = 0 T ( x ) = 0
2

1 1 1


http://press.nctu.edu.tw

12. Assume that R[ x] is the vector space of polynomial with real coefficients, and
W R[ x ] is the subspace spanned by the polynomials:

a1 = 2 4 x 4 x 2 2 x 3 , a 2 = 2 x 2 x3 , a3 = 2 + 4 x 2 3 x 3 .

(a) Prove that a1 , a 2 , and a3 form a basis for W.

(b) If g = g 0 + g1 x + g 2 x 2 + g3 x3 W , then find the components relative to the

ordered basis {a1 , a2 , a3 }.

(c) Let b1 = 1 2 x 2 , b 2 = 4 x 2 + 2 x 3 , b 3 = 3 x3 . Show that {b1 , b 2 , b3 }

also form a basis for W.

(d) Let [ g 0 g1 g 2 ]T be the vector of components of g in (b), which is relative

to the a-basis, and [h0 h1 h2 ]T be the vector of components of g, which is

relative to b-basis in (c). Then find the 3 3 matrix M such that

[ g 0 g1 g 2 ]T = M [h0 h1 h2 ]T .

( 97)

(a) (a)(b)
2 0
4 0
{1, x, x 2 , x3 } a1 = a 2 =
4 2

2 1
2 2 0 2
0 4 0 0
a3 = A =
4 4 2 4

3 2 1 3

(b) g = g 0 + g1 x + g 2 x 2 + g 3 x3 a1 a 2 a3

g0 2 0 2
g 4 0 0 x1
g = = Ax =
1 x x x x
g2 4 2 4 2 1 2 3


x3
g3 2 1 3


http://press.nctu.edu.tw

gi (i = 0,1, 2,3)
2 0 2 g0 2 0 2 g 0 2 0 2 g0
4 0 0 g 0 0 4 g 2 g 0 1 5 g 3 g 0
1
1 0

4 2 4 g 2 0 2 0 g 2 2 g 0 0 2 0 g 2 2 g0

2 1 3 g3 0 1 5 g3 g 0 0 0 4 g1 2 g 0
2 0 2 g0
2 0 2 g0
0 1 5 g g 0 1 5 g3 g0
3 0
0 0 10 g 2 2 g3
0 0 10 g 2 2 g3
2 4
0 0 4 g 2 g 0 0 0 0 2 g 0 + g1 + g 2 g 3
1
5 5
A 3 rank( A) = 3 a1 a 2 a3
W
(b) (a)
2 x1 + 2 x3 = g0
x2 5 x3 = g3 g0
10 x3 = g 2 2 g3
1 1 1 1 1 1
x1 = g 0 + g 2 g3 x2 = g 0 g 2 x3 = g 2 g3
2 10 5 2 10 5
2 4
g W 2 g 0 + g1 + g 2 g3 = 0
5 5

1 0 0
0 0 0
(c) B =
2 4 0

0 2 3

{b1 , b 2 , b3 } W(b)

1 0 0
0 0 6
2 4 0 12
2 1 = 0 [ 0 0 0]
5 5 2 4 0 5 5

0 2 3

b1 b 3 W {b1 , b 2 , b3 } x

a1 x {b1 , b 2 , b3 } W

(d) (c) {b1 , b 2 , b3 } {a1 , a 2 , a3 } W


http://press.nctu.edu.tw

13. Answer the following questions.


(a) Consider a 3 2 matrix A and a 2 5 matrix B. How many possible
dimensions of ker( AB) are there? What are they? Note that ker( AB ) is
the kernel of matrix AB.
(b) Define a linear transformation T from R 22 to R 22 by

1 2 1 2
T (M ) = M M . Find a basis of the image of T with respect
0 1 0 1

1 0 0 1 0 0 0 0
to the standard basis U = , , , .
0 0 0 0 1 0 0 1
( 97)

(a) AB 3 5 AB A A
AB 2 0 rank( AB ) 2
dim ker( AB ) + rank( AB ) = 5 0 5 dim ker( AB ) 2
dim ker( AB ) = 3 4 5

(b) T ( M ) M T (image)

T (range)

a b
M =
c d

a b a b 1 2 1 2 a b
T =
c d c d 0 1 0 1 c d
a 2 a + b a + 2 c b + 2 d 2c 2 a 2 d
= =
c 2c + d c d 0 2c

1 0 0 1 0 0
, ,
0 0 0 0 0 1


2c 2a 2d

0 2c


http://press.nctu.edu.tw

0 1 1 0
,
0 0 0 1

a 0 0 2 0 a

b 2 0 0 2 b
T =
c 0 0 0 0 c

d 0 0 2 0 d


0 0 2 0 2 0 0 2 1 0 0 1
2 0 0 2 0 0 2 0 0 0 1 0
2
0 0 0 0 0 0 0 0 0 0 0 0

0 0 2 0 0 0 2 0 0 0 0 0
0 1

1 0
, 2
0 0
0 1

0 1 1 0
,
0 0 0 1

http://ccjou.twbbs.org/blog/?p=3459

14. Assume M 22 denotes the vector space consisting of all 2 by 2 matrices. That is,

a b a a12
M 22 = , a, b, c, d R . Given two matrices 11 and

c d a21 a22

b11 b12
b b in M 22 , we define their inner product to be
21 22

a11 a12 b11 b12


a , = a11b11 + a12b12 + a21b21 + a22b22 .
21 a22 b21 b22


http://press.nctu.edu.tw

1 1 0 1 1 1 2 0
(a) Assume the set of four matrices , , ,
0 1 2 1 1 0 4 4
span a subspace K in M 22 .
i. Please find the dimension of K.
ii. Please find the largest subspace in M 22 that is orthogonal to K.
(b) Let T : M 22 M 22 be a linear transformation defined as

a b a + b c + d 2b + 3c d a b
T = for any M 22 . Please
c d a + 3c 2a + c + d c d

find the null space of T.


( B98)

(a)
1 1 0 1 1 1 0 1 1 1 0 1 1 0 0 4
0 1 2 1 0 1 2 1 0 1 2 1 0 1 0 3

1 1 1 0 0 0 1 1 0 0 1 1 0 0 1 1

2 0 4 4 0 2 4 2 0 0 0 0 0 0 0 0
4
3
rank K = 3 K
1

1
4 3

1 1
(b) T
1 1 1 1 1 1 1 1
0 2 3 1 0 2 3 1
A=
1 0 3 0 0 0 1 1 2

2 0 1 1 0 0 0 1

rank A = 4 N (T ) = {0}


http://press.nctu.edu.tw

15. Let V be a vector space of continuous functions defined on the interval [0, 2 ]

and = {1 (t ), 2 (t ),, n (t )} be a basis for V.

{
(a) Is the set W = f (t ) V :
0
2
f (t )dt = 0 } a subspace of V? Justify your
answer.
n n
(b) Define T : V V by f (t ) = aii (t ) V , T ( f (t ) ) = ai 1i (t ) V ,
i =1 i =1

where a0 = 1. Prove that T is a linear transformation.


(c) Find bases for both the nullspace of T and the range of T.
( 97)

(a) W = f (t ) V : { 2

0 }
f (t )dt = 0

2 2
f (t ) W g (t ) W f (t )dt = 0 g (t )dt = 0
0 0

2 2 2

0
( f (t ) + g (t ) ) dt = 0 f (t )dt + g (t )dt = 0 + 0 = 0
0

2 2
( cf (t ) ) dt = c
0 0
f (t )dt = 0 W V

(b) T T ( f (t ) + g (t ) ) = T ( f (t ) ) + T ( g (t ) )

T ( cf (t ) ) = cT ( f (t ) )
n n n
f (t ) + g (t ) = aii (t ) + bii (t ) = ( ai + bi )i (t ) T
i =1 i =1 i =1
n n n
T ( f (t ) + g (t ) ) = (ai 1 + bi 1 )i (t ) = ai 1i (t ) + bi 1i (t )
i =1 i =1 i =1

= T ( f (t ) ) + T ( g (t ) )
n n
cf (t ) = c aii (t ) = caii (t ) T
i =1 i =1
n n
T ( cf (t ) ) = cai 1i (t ) = c ai 1i (t ) = cT ( f (t ) )
i =1 i =1

T
V 1 (t )
1


http://press.nctu.edu.tw

n
T ( f (t ) + g (t ) ) = ( ai 1 + bi 1 )i (t ) + 1 (t )
i =2
n n
ai 1i (t ) + 1 (t ) + bi 1i (t ) + 1 (t ) = T ( f (t ) ) + T ( g (t ) )
i =2 i=2
n
n

T ( cf (t ) ) = cai 1i (t ) + 1 (t ) c ai 1i (t ) + 1 (t ) = cT ( f (t ) )
i=2 i =2
T

T ( 0 ) = 0 ( T ( f (t ) f (t ) ) = T ( f (t ) ) T ( f (t ) ) = 0 )
n
T ( 0 ) = ai 1i (t ) + a01 (t ) = 1 (t ) 0
i=2

(c) = {1 (t ), 2 (t ),, n (t )} V
n
T ( f (t ) ) = ai 1i (t ) = 0 ai = 0 i = 0,1, , n 1
i =1

(b) a0 = 1 T
n
T ai 1i (t ) + 1 (t )
i =2

V T V

T


http://press.nctu.edu.tw

1
1
16. Assume T : R R3 2
is a linear transformation with T 0 = ,
0 1

1 1
1.5 1.5
T 1 = , and T 1 = .
0 0.5 1 1.5

a
(a) If we represent any vector x in R as x = b and the vector T(x) in R 2
3

d
as T ( x) = , please find the corresponding matrix A such that
e

a
d
e = A b .
c

(b) Find the range of T.

1 0 1
(c) If we represent any vector x in R as x = x1 0 + x2 1 + x3 0 and
3
1 0 1

1 0
the vector T(x) in R 2 as T ( x ) = y1 + y2 , please find the
1 1

x1
y x .
corresponding matrix M such that 1 = M 2
y2 x3

( B98)

(a) R3 (image) T

1
1
A T 0 =
0 1


http://press.nctu.edu.tw

0 1 1
1.5 1 0.5
T 1 = T 1 T 0 = =
0 0 0 0.5 1 0.5

0 1 1
1.5 1.5 0
T 0 = T 1 T 1 = =
1 1.5 0.5 1
1 0
1 0.5 0
A=
1 0.5 1
(b) Range(T ) = C ( A) A Range(T ) = R 2

1 0 1
1 0
(c) M = 0 1 0 N =
1 1
1 0 1

B A = NBM 1 B = N 1 AM
1 0.5 1
B=
1 1 1

17. Mark each statement True or False. Just write down your answer. There is no
need to specify reasons.
(a) If the columns of A are linear independent, then Ax = b has exactly one
solution for every b.
(b) If U and W are two subspaces of a vector space V, the intersection of U and
W is also a subspace of V.
(c) A square matrix with distinct eigenvalues is diagonalizable.
(d) If two square matrices have the same determinant, then they are similar.

(e) If T is a linear transformation and {u1,,u k } is a linearly independent set

in the domain of T, then {T (u1 ),,T (u k )} is also linearly independent.

( C98)

(a) Ax = b
(b)
(c)


http://press.nctu.edu.tw

(d)
(e) x T ( x) = 0

(b)(c)

18. Let A1 be the matrix representation of a linear transformation from R n to R n

with respect to the basis B1. Let B2 be another basis of R n , and let P be the

transition matrix corresponding to the change of basis from B1 to B2 . What is


the matrix representation of L with respect to the basis B 2 ? Express it in terms
of A1 and P.
( 96)

A1 [ x]B1 = [ y ]B1 [ x ]B1 [ y ]B1 B1

P B1 B2 [ x ]B2 = P [ x]B1 [ y ]B2 = P [ y ]B1

A2 [ x]B2 = [ y ]B2

B1 B2

[ x ]B1 
A1  [ y ]B1
P 1 P
[ x]B2 
A2 [ y ]B2

A2 = PA1P 1

A2 [ x ]B2 = [ y ]B2 A2 P [ x ]B1 = P [ y ]B1 = PA1 [ x]B1 A2 P = PA1 A2 = PA1P 1


http://press.nctu.edu.tw

19. Let L be the linear operator that rotates vectors in R 2 by 45 in the
counterclockwise direction.
(a) Find the matrix representation of L with respect to the natural basis

1 0
{e1 , e2 } , where e1 = , e 2 = .
0 1

(b) Find the matrix representation of L with respect to the ordered basis

1 1
{u1 , u 2 } , where u1 = , u 2 = .
0 1

( 97)

cos sin
(a) R ( ) =
sin cos

1 1 1 1
( )
R 45 = 1 1 0
2

0
45
1
(b) U = {u1 , u 2 } E = {e1 , e2 }

1 1 1 1
P= v = u 2 = v = = 0 u1 + 1 u 2
0 1 1 1

0
U [ v ]U = E
1
0 1
[ v ]E = P = P
1 1
P = [u1 u 2 ]

[ L( v)]E = R [ v ]E U

[ L( v)]E = P [ L( v)]U [ v ]E = P [ v ]U [ L( v)]E = R [ v ]E

P [ L( v) ]U = RP [ v ]U [ L( v)]U = P 1 RP [ v ]U P 1 RP


http://press.nctu.edu.tw


1
1 1 1 1 1 1 1 1 1 0 2
P RP = 1 1 0 1 =
0 1 2 2 1 2

20. Label the following statements as being true or false.

(a) If the reduced row echelon form of [A b ] contains a zero row, then

Ax = b has infinitely many solutions.


(b) A function from R n to R m is uniquely determined by its images of the
standard vectors in R n .
(c) Let A be an m n matrix. Then the rank of A is n if and only if the
equation Ax = b has at most one solution for each b in R m .
(d) Let A, B and C be any matrices such that the product ABC is defined. Then
rank( ABC ) rank B.

(e) Let A = [a1 a 2  a n ] be a square matrix and b be a linear

combination of a1 , a 2 , , a n . Then det A = det [a1 + b a 2  a n ] .

(f) If V and W are subspaces of R n having the same dimension, then V=W.
(g) Every column of A can be uniquely expressed as a linear combination of the
pivot columns of A.
(h) Let V be a subspace of R n and W be its orthogonal complement. If v is a
vector in V and w is a vector in W, then v iw = 0.
(i) Let S be a set containing n linearly independent eigenvectors of an n n
symmetric matrix. Then S forms an orthogonal basis for R n .
(j) A matrix representation of a linear operator on M mn is an m n matrix.
( 98)

(a)

Ax = b [A b ]

1 0 1

[ A b ] 0 1 1 [ A b ]
0 0 0


http://press.nctu.edu.tw

Ax = b
(b) R n R m m n
A R n (image)

a a c 1
f (x) = 11 12 x + 1 f
a21 a22 c2 0

0
f f
1
(c)
http://ccjou.twbbs.org/blog/?p=1501
(d)

http://ccjou.twbbs.org/blog/?p=1766

(e) A = [a1 a 2 ] b = a1

det [a1 + b a 2 ] = det [ 2a1 a 2 ] = 22 det [a1 a 2 ] = 4 det A

(f)
(g)
http://ccjou.twbbs.org/blog/?p=1388
(h)
http://ccjou.twbbs.org/blog/?p=1861
(i)
R n

(j) M mn m n
T : M mn M mn m n
( mn) ( mn)


http://press.nctu.edu.tw

Chapter 4
Orthogonality


http://press.nctu.edu.tw

Let a1 = [1,1, 0] , a 2 = [ 2,3, 0] , and b = [ 4,5, 6 ] . Find the projection vector


T T T
1.

of b onto the plane that is spanned by the vectors a1 and a 2 .


( 95)

1 2
1 3 ( z )
0 0

1 0 4
0 1 5

0 0 0

2. L is the line ax + by = 0. P = ( x, y ) is an arbitrary point of the xy plane. If

Q = ( x1 , y1 ) is the foot of the perpendicular from P to L, find a 2 2 matrix A

x x
such that 1 = A .
y1 y

( 96)

x
A p = L L = span {v}
y

b a x vT p vvT
v = v 1 = T v = T p = Ap
a b y1 v v v v

vvT 1 b 2 ab
A = = 2 2
v v a +b
T
ab a2


http://press.nctu.edu.tw

vT p v ( vT p ) ( vvT ) p vvT
T v = = = T p
v v vT v vT v v v

3. Answer the following problems dedicated to the orthogonality and its


least-squares application.

5 1
3
(a) There are three vectors in real space R : u1 = 2 , u 2 = 2 ,

1 1

1
y = 5 . Let W be a subspace spanned by u1 and u 2 . Find a specific
10

point in W, which is closet to y.

1 5
(b) Find a least-squares solution of the equation Ax = b for A = 3 1
2 4

4
and b = 2 .
3

( 95)

5 1 5 1
(a) W u1 = 2 u 2 = 2 A = 2 2

1 1 1 1

1
y = 5 W ( A C ( A) )
10

Ax y Ax C ( A) AT ( y Ax ) =0


http://press.nctu.edu.tw

AT Ax = AT y (normal equation) A
rank( A) = rank( AT A) AT A

( ) AT y p = Ax = A ( AT A ) AT y
1 1
x = AT A


y u1 u 2
1
uT u 0 u1T 1 u1T u1 0 u1T y
p = [u1 u2 ] 1 1 y = [ u1 u ]
2
0 uT2 u 2 uT2 0 1 uT2 u 2 uT2 y

u1T y uT2 y
p = u1 + u 2
u1T u1 uT2 u 2

5 1 1
15 21
p = 2 + 2 = 8
30 6
1 1 4

0
e = y p = 3 e u1 u 2
6

(b) (a) A = [a1 a 2 ]

a1T b aT2 b
p = a1 + a 2
a1T a1 aT2 a2

1 5 1
a1T b a1T a1 2 7 2 7
x = T T = p = 3 1 = 1
a 2 b a 2 a 2 1 7 17
2 4 0

http://ccjou.twbbs.org/blog/?p=3631


http://press.nctu.edu.tw

Let R 4 have the Euclidean inner product. Express w = [ 1, 2, 6, 0] in the form


T
4.

w = w1 + w 2 , where w1 is in the space W spanned by u1 = [ 1, 0,1, 2] and


T

u 2 = [ 0,1, 0,1] , and w 2 is orthogonal to W.


T

( 95)

u1 = [ 1, 0,1, 2] u 2 = [ 0,1, 0,1]


T T

1 0
0 1
A= w1 w = [ 1, 2, 6, 0] A
T

1 0

2 1

P = A ( AT A ) AT A
1

5 1 5 1
1 2 1
1 1 1 9
w1 = Pw = w 2 = w w1 = =
4 5 6 4 5 4 19

9 0 9 9

w1T w 2 = 0

5. The point r = ( x, x, x ) is on a line through (1,1,1) . And, the point


s = ( y + 1, 2 y,1) is on another line.
2
(a) Choose x and y to minimize the squared distance r s .

2
(b) Find the minimum value of r s .

( 95)

1 1 y + 1

r = ( x, x, x ) 1 A = 1 s = ( y + 1, 2 y,1) b = 2 y
1 1 1


http://press.nctu.edu.tw

Ax = b

( b Ax ) Ax AT Ax = AT b

1 y + 1
[1 1 1] 1 x = [1 1 1] 2 y x = y +
2
3
1 1

y + 1 1 1 3
2
e = b Ax = 2 y y + 1 = y 2 3

3
1 1 y + 1 3

T
13 13 2

e = y 2 3
2 y 2 3 = 2 y 2 2 y + 2 d e = 4 y 2 y = 1
3 dy 2
y + 1 3 y + 1 3

2 2 1 7
r s = e x = (a)(b)
6 6

a 1 2 1 2

6. Find a, b, and c such that the matrix A = b 1 6 1 6 is orthogonal.

c 1 3 1 3

( 95)

(cross product)

a 1 2 1 2 a

b 1 6 1 6 b

c c
1 3 1 3

1 2 1 6 1 3

1 2 1 6 1 3

1 2 1 6 1 3 1 2 1 6 1 3 1 0 0

1 2 1 6 1 3 0 2 6 2 3 0 1 2


http://press.nctu.edu.tw

0 a 0

2 b = 2 3
1 c 1 3

1 1 1 4 1
2 1 1 5 5
7. Assume A = .
1 2 1 1 3

1 1 2 0 2
(a) What is the rank of this matrix?
(b) Find the nullspace of A.
(c) Find an orthonormal basis for the column space of A.

(d) Find the projection of y = [1 2 4 8]


T
onto the column space of A.

( 96)


(a)
1 1 1 4 1 1 0 0 3 2
2 1 1 5 5 0 1 0 1 2

1 2 1 1 3 0 0 1 2 1

1 1 2 0 2 0 0 0 0 0
3 rank( A) = 3

3 2
1 2

(b) (a) N = 2 1 A

1 0
0 1

N
(c) A 123
Gram-Schmidt


http://press.nctu.edu.tw

1 1
2 1
u1 = a1 = a 2 a1 u 2 = a 2 =
1 2

1 1

1 1 1 9
1 4 2 2 1 1 3

u1T a3 uT2 a3
u 3 = a3 T u1 T u 2 = =
u1 u1 u2 u2 1 7 1 7 2 7 7

2 1 1 8

1 1 9
2 1
1 1 1 3
q1 = q 2 = q 3 =
7 1 7 2 203 7

1 1 8
(d) (c) y

p = ( y T q1 ) q1 + ( y T q 2 ) q 2 + ( y T q 3 ) q 3

9 17 21
y T q1 = yT q2 = y T q3 =
7 7 203
1 1 9
2 1
9 17 3 3
p = +
7 1 7 2 29 7

1 1 8


http://press.nctu.edu.tw

8. Let V = span {(2, 0, 1,1)T , (1,1, 0,1)T } .

(a) Find an orthonormal basis for V.


(b) Let b = (1,1, 3,1)T . Use your answer to (a) to find the projection p of b
onto V.
2 1
0 1
(c) Let A = . Find the least-squares solution of Ax = b.
1 0

1 1
( 96)

(b)(c)

2 1 2
0 1 0
(a) a1 = a 2 = Gram-Schmidt v1 = a1 =
1 0 1

1 1 1
1 2 0
1 3 0 1

aT2 v1
v 2 = a 2 T v1 = =
v1 v1 0 6 1 1 2

1 1 1 2
2 0
0
1 1 2
q1 = q2 =
6 1 6 1

1 1

(b) (a) {q1 , q 2 } V b = (1,1, 3,1)T V


2 0 2
0
6 0 2 0
p = ( bT q1 ) q1 + ( bT q 2 ) q 2 = + =
6 1 6 1 1

1 1 1
1
(c) (b) x =
0


http://press.nctu.edu.tw

1 1
9. Let A = 1 2 and S be the subspace of R 3 spanned by the column vectors
2 1

of A.
(a) Find an orthogonal basis for S , the orthogonal complement of S.

(b) Let P = A ( AT A) AT , if x R 3 and x S S , show that Px S .


1

( 97)

(a) S A S = C ( A)
S = C ( A) = N ( AT )C ( A) = N ( AT )
C ( AT ) = N ( A) N ( AT ) AT
AT

1 1 2 1 1 2 1 1 2 1 0 1
1 2 1 0 3 3 0 1 1 0 1 1

1 1

N = 1 S 1

1 1

(b) Px = A ( AT A ) AT x Px = A ( AT A ) AT x = Ay
1 1


x Px A Px A
S A Px S
C ( P) C ( A) x C ( A) x = Az

( )
1
Px = A AT A AT Az = Az = x C ( A) C ( P ) C ( A) = C ( P )

x S S


http://press.nctu.edu.tw

10. In the inner product space R 3 with the inner product function u, v = uT Dv

for all u and v in R 3 , where D = [e1 4e2 e3 ] and ei is the ith standard

vector of R 3 , find the least-squares approximation x of the following problem:

1 0 1
0 1 x = 2 .

1 1 1

( 95)

uT v = uT Iv I

1 0 0
I D D = 0 4 0 u, v = uT Dv
0 0 1

2
u D
= u , u = u T Du

1 0 1
A = 0 1 b = 2 b Ax D
2

1 1 1

ai , b Ax = aTi D(b Ax ) = 0 i = 1, 2

AT D (b Ax ) = 0 AT DAx = AT Db
A D

2 1 0
x = ( AT DA) AT Db AT DA =
1
AT Db =
1 5 9

1 5 1 0 1
x = =
9 1 2 9 2


http://press.nctu.edu.tw

11. u and v are orthogonal if ui v = 0. The S is the set of all vectors in R n that
are orthogonal to every vector in S. Consider the set

x1

S = x2 R : x1 x2 + x3 = 0 . Choose the following statement which is
3

x
3
correct.

1
(a) 1 S

0

1
(b) 1 S

1

(c) S is a subspace of R 3 and dim S = 1.

1 1 3

(d) Let 0 = w + z such that w S and z S , then z = 1 3 .

0 1 3

1 1

(e) 0 , 1 is a basis for S.
1 1

( 97)

x1

S = x2 R : x1 x2 + x3 = 0 (constraint
3

x
3

1

way) S = span 1

1

(a)(b)
dim S = 1 dim S = 3 dim S = 3 1 = 2 (c)


http://press.nctu.edu.tw

1 1 1 1

0 S = span 1 z b = 0 v = 1

0 1 0 1

1
bT v 1
z = T v = 1 (d)
v v 3
1

1 1 1

0 , 1 1
1 1 1

1 1 1

1 (e) 0 , 1
1 1 0

(d)

1 0 2
12. Let A = 0 1 1 .
1 2 0

(a) Use the Gram-Schmidt process to factor A into a product QR, where

Q = [q1 , q 2 , q3 ] is a matrix with orthonormal column vectors {q1 , q 2 , q3 }

and R is an upper triangular matrix with positive diagonal entries.

(b) If x = 2q1 + 2q 2 + q3 and y = 5q1 + q 3 , determine the vector norm x 2

and the inner product x, y .

( 97)

(a) Gram-Schmidt A = [a1 a2 a3 ]

{q1 , q 2 , q3 }


http://press.nctu.edu.tw

1 0 1 1
v1T a 2 2
v1 = a1 = 0 v 2 = a 2 T v1 = 1 0 = 1
v1 v1 2
1 2 1 1

2 1 1 1
v1T a3 vT2 a3 2 1 2
v 3 = a3 T v1 T v 2 = 1 0 1 = 2
v1 v1 v2 v2 2 3 3
0 1 1 1

1 1 1
1 1 1
q1 = 0 q2 = 1 q3 = 2
2 3 6
1 1 1

QR

QR A Q = [q1 q 2 q3 ]


a1 = ( q1T a1 ) q1
a 2 = ( q1T a 2 ) q1 + ( qT2 a 2 ) q 2
a3 = ( q1T a3 ) q1 + ( qT2 a3 ) q 2 + ( qT3 a3 ) q3

q1T a1 q1T a 2 q1T a3



A = [a1 a 2 a3 ] = [q1 q 2 q3 ] 0 qT2 a 2 qT2 a3 = QR
0 0 qT3 a3

1 2 1 3 1 6 2 2 2

Q = 0 1 3 2 6 R = 0 3 1 3

1 2 1 3 1 6 0 0 4 6

2
(b) x = 2q1 + 2q 2 + q3 = Q 2 = Qb Q
1

QT = Q 1 x = ( Qb ) (Qb ) = bT QT Qb = bT Ib = bT b
2 T

5
x = 9 x = 3 y = 5q1 + q3 = Q 0 = Qc
2


http://press.nctu.edu.tw

x, y = xT y = ( Qb ) ( Qc ) = bT QT Qc = bT c = 11
T

http://ccjou.twbbs.org/blog/?p=4110

13. Answer the following questions.

(a) Prove that if A and B are two matrices with m rows, and N ( AT ) N ( BT ) ,

then R ( B ) R ( A).

(b) Let V be a subspace. Show that (V ) = V .


( 97)

(a)
R ( A) A (range) A

C ( A ) N ( AT ) N ( BT ) dim N ( AT ) dim N ( BT )

dim N ( AT ) + rank( A) = m

rank ( A) rank( B) rank ( A) = dim R( A)

dim R ( A) dim R( B) R ( B) R( A)

1 0
dim R ( A) dim R( B) R ( B) R( A) A = 0 1
0 0

0
dim R( A) = 2 B = 0 dim R( B) = 1 R ( B ) R( A)
1

R( A) N ( AT ) R m

() R( A) N ( AT ) = {0} R m


http://press.nctu.edu.tw

x x = y + z y R( A) z N ( AT )

x R m x = y + z

x = y + z y R( A) z N ( AT ) y + z = y + z

y y = z z y y R ( A) z z N AT ( )
R( A) N ( AT ) = {0} y y = z z = {0}

{v1 , , v r } {v r +1 , , v m } R( A) N ( AT ) {w1 , , w s }

{w s +1 , , w m } R( B) N ( BT )

x R m {v1 , , v m }

x = c1 v1 +  + cr v r + cr +1 v r +1 +  + cm v m

{w1 , , w m } x = d1w1 +  + d s w s + d s+1w s +1 +  + d m w m

d1w1 +  + d s w s = x ( d s +1w s +1 +  + d m w m )
= c1 v1 +  + cr v r + cr +1 v r +1 +  + cm v m ( d s +1w s +1 +  + d m w m )

N ( AT ) N ( BT ) {v r +1 , , v m }

{w s +1 , , w m } r s

d1w1 +  + d s w s = c1 v1 +  + cr v r + es +1w s +1 +  + em w m {w1 , , w s }

{w s +1 , , w m } {w1 , , w s }

{v1 , , v r } R ( B) R( A)

(b) x V y V xT y = 0 y V z (V )

y T z = 0 z T y = 0 y V z V z (V )


http://press.nctu.edu.tw

(V ) = V

14. Let T be a linear operator on P2 defined by

T ( f ( x )) = f (0) + f ( x ) + f ( x ) + f (1) x 2 .

(a) Find [T ]B , where B is the standard basis for P2 .

(b) For f ( x ) = a0 + a1 x + a2 x 2 , find T 1 ( f ( x )) .

( 98)

(a) P2 {1, x, x 2 } T ( f ( x ) )

( ( )) = 2x + 2x
T ( f (1) ) = 2 + x 2 T ( f ( x ) ) = 1 + x + x 2 T f x 2 2

2 1 0

[T ]B = [T ( f (1)) ]B
[T ( f ( x))]B 2
T ( f ( x )) B = 0 1 2
1 1 2

2 1 0 a0
(b) 0 1 2 y = a1

1 1 2 a2

0 1 1
2 2
[T ]B [T ]B
1
= 1
1 2 1 2 1

1 1
T 1 ( f ( x) ) = ( a1 + a2 ) + ( a0 + 2a1 2a2 ) x + a0 a1 + a2 x 2
2 2


http://press.nctu.edu.tw

15. A subset of R n is called orthogonal set if every pair of distinct vectors in the set
is orthogonal. An orthogonal projection of v onto a subspace W is defined as a
vector, w W such that v = w + z , where z W . Which statements in the
following are correct?
(a) Any orthogonal set of nonzero vectors is linearly independent.
(b) Every subspace has an orthogonal basis.

(c) For any matrix A, ( Row A) = Null A.

(d) Let W be a subspace of R n and v be a vector in R n . Among all vectors in


W, the vector closest to v is the orthogonal projection of v onto W .
(e) For any subspace W of R n , dim W + dim W = n.
( 97)

(a)

(b) {0} dim {0} = 0

dim W > 0
(c) x N ( A) Ax = 0 x C ( AT )
(d) v W W
(e)
(a)(c)(e)

16. Give an example of a linear map L from R 5 to R 2 whose null space equals

{[ x
1 }
 x5 ] R 5 : x1 = 3 x2 , x3 = x4 = x5 , or prove that no such linear maps
T

exist.
( 98)

2 5 A N(A) x
x1 = 3 x2 x3 = x4 = x5
3 3 0 3 0
1 0 1 0

x = = 0 + 1 0 , 1 dim N ( A) = 2

0 1 0 1
0 1 0 1


http://press.nctu.edu.tw

rank A = 5 dim N ( A) = 5 2 = 3 L
R 2

17. Assume M 22 denotes the vector space consisting of all 2 by 2 matrices. That is,

a b
M 22 = , a, b, c, d R . On the other hand, assume C[1,1] denotes
c d
the vector space of all continuous function f ( x) defined over x [ 1,1]. In
C[ 1,1], the inner product of two continuous functions p ( x ) and q ( x) is
1
defined as p( x), q( x) = p( x)q( x)dx. Let T : M 22 C[1,1] be a linear
1

transformation defined as

a b
= ( a + b + c + d ) x + ( a + b c d ) x + ( a b + c d ) x + ( a + 2b c )
3 2
T
c d

a b
for any M 22 .
c d

(a) Find the nullspace of the transformation T.


(b) Assume Q denotes the range of this transformation. What is the dimension
of Q?

(c) Assume W is a subspace of C[ 1,1] and is spanned by p1 ( x ) = x 2 and

1 2
p2 ( x) = x, with x [1,1]. Find the projection of T on W.
2 1
( 97)

2


(a) T (
)(
)
2 1
5 3 T 11x 5 x + 3 x 1
3 2


http://press.nctu.edu.tw

T : M 22 C[1,1] T
0
(kernel)


M 22
( 2 2 ) 4 1 4
4

B = { x3 , x 2 , x,1} T

a a 1 1 1 1 a
b 1 1 1 1 b
b
T = A =
c c 1 1 1 1 c

d d 1 2 1 0 d

a a

b b
T T

c c

d d B

()

a 1 1 1 1 2 11

T b = 1 1 1 1 1 = 5
c 1 1 1 1 5 3

d B 1 2 1 0 3 1
1 0 0 1
0 1 0 1
A
0 0 1 1

0 0 0 0

1 1
1
1

1 1

1 1

1 1
1 1 T 0


http://press.nctu.edu.tw


(b) 2
T (range) Q
dim Q = rank(T ) = 4 dim ker(T ) = 4 1 = 3 (a)
dim ker(T ) = 1
1 1 1 1 1 6
1 2 1 1 1 1 2 0

(c) T
2 1 = 1 1 1 1 2 = 0
B

1 2 1 0 1 3
b( x) = 6 x 3 + 3 3 b( x) 2
sx 2 + tx

1
1 1 x4
p( x), q ( x) = p( x)q( x)dx. p1 ( x), p2 ( x) = x 3dx = =0
1 1 4 1

W p1 ( x) = x 2 p2 ( x) = x

p1 ( x), b( x) p2 ( x), b( x)
p1 ( x) + p2 ( x)
p1 ( x), p1 ( x) p2 ( x), p2 ( x)
1 1

=
1
x 2 (6 x 3 + 3)dx
(x ) +
2 1
x(6 x3 + 3)dx
( x)
1 1
1
x 4 dx 1
x 2 dx
5 2 1
6 x 3x
1 +
x6 + x3 5 2 18
=
5 1
1
(x )+
2
3 1
1
( x ) = 5x2 +
5
x
x x
5 1
3 1

18. Find an orthogonal basis for the subspace C[0,1] that is spanned by {1, et , e t } .

Note: The definition of an inner product for f and g in C[a,b] is


b
f , g = f (t ) g (t )dt.
a

( 96)


http://press.nctu.edu.tw

a1 = 1 a 2 = et

a 3 = e t Gram-Schmidt v1 = a1 = 1

1 1
e dt 1 = e
t
a2 , v1 et e 1 t
v 2 = a 2 v1 = e t
0
1
t
1
0
= et = e e +1
v1 , v1
1dt
0
t0 1

e ( e + 1 e ) dt ( e
1 1
e t dt t t
a3 , v1 a3 , v 2
v 3 = a3 v1 v2 = e t
0
0 t
+1 e )
( e + 1 e ) dt
1 1 2
v1 , v1 v2 , v2

t
dt
0 0
1
3 e e
= e t + e 1 1 +
1 2 3
et + 1 e ( )
e 2e +
2 2

19. The set of polynomials with real coefficients is a vector space denoted by P ( R ).
Let n be a nonnegative integer, and let Pn ( R ) consist of all polynomials in
P ( R ) having degree less than or equal to n. Let V = P( R ) with inner product
1
f ( x), g ( x) = f (t ) g (t )dt , and consider the subspace P2 ( R ) with the
1

ordered basis = { x 2 , x,1} . Use the Gram-Schmidt process to replace by an

orthonormal basis {v1 , v 2 , v3 } for P2 ( R ) in the order of x 2 x 1.

( 97)

x 2 x 1
1

u1 = x u 2 = x
2
x2 , x
x 2
= x
1
x 3 dx
x 2
1
x2 , x2

4
x dx
1

1 2 1
x 2 n dx = x 2 n +1dx = 0 u 2 = x
1 2n + 1 1


http://press.nctu.edu.tw

1 1 2
u3 = 1
x 2 ,1
x
2 x,1
x = 1
1
x 2 dx
x 2

1
xdx
3 5
x = 1 x2 = 1 x2
1 1
x2 , x2 2
x, x
x 4 dx 3
2
x dx
1 1
5

1 10 2
v1 = x2 = x
1 2

1
x 4 dx

1 6
v2 = x= x
1 2
1
x 2 dx

1 5 2
v 3 = 1 3 x
1 5 2
2

1
1 x dx
3

2
1 5 2 1 10 2 25 4 10 2 25 2 8
1 1 3 x dx = 1 1 3 x + 9 x dx =2 3 3 + 9 5 = 9
3 2 5 2
v3 = 1 x
4 3

20. Consider the inner product space C[0, 1] which is the set of all functions that
have a continuous first order derivative on [0, 1]. The inner product of two
1
functions f ( x) and g ( x ) is defined by f , g = f ( x) g ( x)dx.
0

(a) Use the Gram-Schmidt process to find an orthonormal basis E for the
subspace S spanned by the vectors 1 and x 2 .
(b) Find the best least squares approximation to the function x on the
interval [0, 1] by a function in S.
( 97)

(a)
u1 = 1
1

1
x3
1 x dx 1 = x
2
u1 , x 2 3 1
u2 = x 2
u1 = x 2
0
1
2
1
0
= x2
u1 , u1
11dx
0
x0 3


http://press.nctu.edu.tw


u1 u 2
u
q1 = 1 = u1 = 1
u1

1 1 1
x2 x2 x2
u
q2 = 2 = 3 = 3 = 3 = 3 5 x2 1

u2 2 1
2 1
4 2 3
1
x 2x
5
x 3

0 x 3 dx + 45
5 9 90

3 5 2 1
E 1, x
2 3

(b)

A = [q1 q 2 ] b = x

p = A ( AT A ) AT b A
1

1 0
AT A = = I
0 1

qT b
( ) ( )
p = AAT b = [q1 q 2 ] 1T = q1T b q1 + qT2 b q 2 qT b
q 2 b

q, b p = q1 , b q1 + q 2 , b q 2

1 x3 2 2
q1 , b = 1 xdx = =
0 3 3
2 0
1

13 5 1 3 5 x7 2 1 x3 2 3 5 2 2 2 5
q2 , b = x xdx =
2
7 3 = =
0 2
3 2 3 2 7 9 21
2 2 0
2 2 5 3 5 2 1 5 2 3
p = 1 + x = x +
3 21 2 3 7 7


http://press.nctu.edu.tw

21. Consider the vector space C [ 0,1] with inner product defined by

f , g = f ( x ) g ( x )dx, where C [ 0,1] denotes the set of all real-valued


1

functions that are defined and continuous on the closed interval [0,1].
(a) Use the Gram-Schmidt process to find an orthonormal basis for the
subspace S spanned by 1 and x.
(b) Find the best least squares approximation to e x on the interval [0,1] by a
function in S.
( 98)

(a) p1 = 1 p2 = x v1 = p1 = 1
1
1
1 2
x
v2 = p2
p2 , v1
v1 = x
xdx 1 = x
0 2 0 1
= x
1 1
v1 , v1
dx
0
x0 2

v1
u1 = =1
v1 , v1

v2 1 1 1
u2 = = x = 12 x
v2 , v2 1 12 2 2

(b) (a) v = c1u1 + c2u2 e x S u1

1 1
u2 c1 = e x ,1 = e x dx = e x = e 1
0 0

1 1 1
c2 = e x , 12 x = 12 e x x dx
2 0
2
1
1 x 1 2 1 x 1 3
0 e x 2 e dx = e x e 2 e 0 = 2 e + 2
x x

1
v = e 1 + 6 ( e + 3) x = 6 ( e + 3) x + 4e 10
2


http://press.nctu.edu.tw

22. For any matrix A, let N ( A) denote its nullspace. In the real space R n ,
1/ 2
consider the inner product x, y = x1 y1 +  + xn yn and 2-norm x = x, x

for every vectors x = [ x1  xn ] and y = [ y1  yn ]


T T
in R n . Suppose S

is a subspace of R n . Let S be the orthogonal complement of S in R n . For


the following matrix A,
(a) find a basis for N ( A) , and

(b) check if x = [ 0 0 1 0 0]
T
is a vector with the smallest 2-norm

satisfying Ax = [1 2 2 1]
T
and explain why.

3 3 1 3 3
2 4 2 4 2
A=
0 3 2 3 0

1 1 1 1 1
( 96)

(a) N ( A) = C ( AT ) A

3 3 1 3 3 1 0 1 3 0 1
2 4 2 4 2 0 1 2 3 1 0

0 3 2 3 0 0 0 0 0 0

1 1 1 1 1 0 0 0 0 0

1 0

0 1

1 3 , 2 3
0
1

1 0

0
0 1
2
(b) x = 1 Ax = b = R n
2
0
0 1

x x = x N + x R x N N ( A) x R C ( AT )


http://press.nctu.edu.tw

Ax = Ax N + Ax R = 0 + Ax R = b x R

2
x x = xT x x = x N + x R

x = ( x N + x R ) ( x N + x R ) = x N
2 T 2 2
+ 2xTN x R + x R

2 2 2 2
x N x R x = x N + xR x R x N = 0

x C ( AT )

1 0
0 1

B = 1 3 2 3 x R = B ( BT B ) BT x
1


0 1
1 0

3 0
6 0
1
x R = 5 1
23
6 0
3 0

x C ( AT )

1 0

0 1

(a) 1 3 , 2 3
0
1

1
0

http://ccjou.twbbs.org/blog/?p=1857


http://press.nctu.edu.tw

23. Consider the two sets of linear equations

x1 2 0 1
2 1 1 1 y1
0 1 1 x2 = 0 and 1 1 y = 0 .
x 1 1 2 1
3

(a) For the inconsistent set, find the least squares solution.
(b) For the consistent set, find the real-valued solution with the minimal

two-norm (the two-norm of a vector w = [ w1  wn ] R n is defined to


T

be w 2 = w12 +  + wn2 ).

( C98)

(a)
AT Ay = AT b A y = ( AT A)1 AT b
1 6
y =
1 2
(b)

1 2
Ax = b x = 0 x A
0

2
T 11
A x = A ( AA ) Ax = 1
T T

6
1


http://press.nctu.edu.tw

24. Let A be a 4 5 matrix. After a series of elementary row operations, we obtain


1 0 4 0 0
0 1 2 0 1
a matrix B = from A.
0 0 0 1 1

0 0 0 0 0
(a) Find matrix A if the 1st, 3rd, and 5th columns of A are (1, 0,1, 0)T ,
(0, 4, 4, 4)T , and (1, 1, 0, 2)T , respectively.
(b) If W is generated by the column vectors of A, find an orthonormal basis for
A.
(c) For a vector x = (1, 1,3,3)T , find the shortest distance from x to W and the
corresponding closest vector to x on W.
(d) For a linear equation system Ax = b, find a basis for the vector space
spanned by the solutions of its homogeneous system.
(e) Following (d), if ( 4, 0,1,3, 1) is in the systems solution set, find its
minimal solution.
( 96)

1 0 1
0 a 4 a 1
(a) A = [a1 a2 a3 a4 a5 ] = 2 4
1 4 0

0 4 2
a 2 a 4 A

4 0
2 1

B N = 1 0

0 1
0 1

Ax = 0

4
2

[a1 a2 a3 a4 a5 ] 1 = 0 4a1 + 2a 2 + a3 = 0

0
0


http://press.nctu.edu.tw

0
2 1
2
1
a 2 = ( 4a1 a3 ) = [a1 a 2 a3 a4 a5 ] 0 = 0
2 0
1
2 1

3
1
a 2 a 4 + a5 = 0 a 4 = a 2 + a5 =
0

0
(b) W A W A
124 A 124

{a1 , a 2 , a 4 } Gram-Schmidt


1 2 1 1
0 T 2 2
u a 2 0
u1 = a1 = u 2 = a 2 1T 2 u1 = =
1 u1 u1 0 2 1 1

0 2 0 2
3 1 1 1
T T 1 0 0
u a u a 3 5 2
u3 = a 4 1T 4 u1 T2 4 u 2 = =
u1 u1 u2 u2 0 2 1 10 1 1

0 0 2 1
1 1 1
0 2
1 1 1 0
q1 = q2 = q3 =
2 1 10 1 3 1

0 2 1
(c) x A p(b)

Q = [q1 q 2 q3 ]

4 3
2
p = QQ x p =
T
8 3

7 3
15
xp =
3
(d) A


http://press.nctu.edu.tw

4 0 4 0
2 1
2 1

(a) N = 1 0 1 , 0
0 1
0 1
0 1 0 1

(e) (minimal solution)



4 4 0
0 2 1

(general solution) y = 1 + 1 + 0

3 0 1
1 0 1

4 4 0
0 2 1

1 1 0

3 0 1
1 0 1

b Ax

x = ( AT A) AT b
1

4
0
21 2 4 2 1 0 0 1
1

= 2 3 0 1 0 1 1 1 = 2

3
1

4 4 0 0
0 2 1 0

1 1 + 2 0 = 0 B

3 0 1 1
1 0 1 1


http://press.nctu.edu.tw

25. Which statements are correct?


(a) Let u1 = ( 1, 2,1), u 2 = (1,1, 2), v = (10, 5,10), and S = span(u1, u 2 ).
17 30
The (shortest) distance between v and S is .
7
(b) For the same setting in the previous question, the least square solution of the
11 4
system x1u1 + x2u 2 = v is x1 = and x2 = .
7 7

(c) Let V be an inner product space, and u1, u 2 denote the inner product of

any two vectors u1, u 2 V . If B = { v1, v 2 ,, v n } is an ordered basis of V,

then for any vector u V , the coordinate of u can be given by

[u ]B = u, v1
T
u, v 2  u, v n .

( 98)

1 1
(a) A = 2 1 v S = C ( A) v
1 2

S = N ( AT ) A
1 2 1 1 2 1 1 0 5 3
1 1 2 0 0 1 1 3 N ( A )
T

3 1

5
u = 1 v S = N ( AT )
3

5 5
uT v 85 17 17
p = T u = 1 = 1 p = 35
u u 35 7 7
3 3


http://press.nctu.edu.tw

10 5 15
1 11
(b) Ax = v p = 5 1 = 18 x =
17 1
7 7 7 4
10 3 19

(c) B = { v1, v 2 ,, v n }

26. The owner of a rapidly expanding business finds that for the first five months of
the year the sales (in thousands) are $4.0, $4.4, $5.2, $6.4, and $8.0. The owner
plots these figures on a graph and conjectures that for the rest of the year the
sales curve can be approximated by a quadratic polynomial. Find the
least-squares quadratic polynomial fit to the sales curve, and use it to project the
sales for the twelfth month of the year.
( 95)

(curve fitting) y t

y (t ) = a0 + a1t + a2t 2

4 = a0 + a1 + a2
4.4 = a0 + 2a1 + 4a2
5.2 = a0 + 3a1 + 9a2
6.4 = a0 + 4a1 + 16a2
8.0 = a0 + 5a1 + 25a2

1 1 1 4.0
1 2 4 4.4

Ax = b A = 1 3 9 b = 5.2

1 4 16 6.4
1 5 25 8.0

a0
x = a1
a2

x = A ( AT A ) AT b
1


http://press.nctu.edu.tw

4
x = 1 5 y (t ) = 4 t + t 2 12
1 1
5 5
1 5

12 1
y (12) = 4 + (12)(12) = 30.4
5 5

http://ccjou.twbbs.org/blog/?p=3809

27. Let Pm ( R ) denote the inner-product space of all polynomials with real
coefficients and degree at most m, with the inner product defined by
1
p, q = p( x )q( x )dx, p, q Pm ( R ). Apply the Gram-Schmidt procedure to
0

{
the basis 1, x, x 2 } to produce an orthonormal basis of P2 ( R ).

( 98)

p1 = 1 p2 = x p3 = x 2 v1 = p1 = 1


1
1
1 2
x
v2 = p2
p2 , v1 xdx 1 = x
v1 = x 0 1
2 0
= x
1
1
v1 , v1
0
dx x0 2


1 1
p ,v p ,v
1
x 2 dx 0
x 2 x dx
2 1
v3 = p3 3 1 v1 3 2 v2 = x 2 0 1 1 2
x
2
v1 , v1 v2 , v2
0 dx 1 1
0 x 2 dx
1 1 1 1 1
x 3dx x 2 dx
1


2
x dx 1 1 4 6 1
= x2 0
0 2 0 = x 2
2
12 x x
1
1
1 2 u du 2 3
2

12
1
= x2 x +
6


http://press.nctu.edu.tw

v1
u1 = =1
v1 , v1

v2 1 1 1
u2 = = x = 12 x
v2 , v2 1 12 2 2

v3 1 2 1
u3 = = x x+ 6
v3 , v3 1
2
1
x x + dx
2
0 6


2
2
1 1 1 4 2 1 1 1 1 4 1 1 1
0 + = x 2 x + x x + dx = + + =
4 3
x x dx
6 0
3 3 36 5 2 9 6 36 180

1
q3 = 6 5 x 2 x +
6

http://ccjou.twbbs.org/blog/?p=3101


http://press.nctu.edu.tw

28. For any matrix A, let N(A) denote its null space. In the real space R n , define the
2 1/ 2
inner product x, y = x1 y1 +  + xn yn and 2-norm x = x, x for all

vectors x = [ x1  xn ] and y = [ y1  yn ] . Suppose S is a subspace of


T T

R n . Let S be the orthogonal complement of S in R n with respect to the

inner product , . Consider a real 4 4 matrix A with N(A) spanned by the

2 1 3

0 2 1
set , , .
2 0 4
1 1 1

(a) What is the rank of A?


(b) Find an orthonormal basis B for N ( A) .
(c) With the above information, judge which one of the following three
conditions can uniquely determine the matrix A, and find the unique A
under that condition:
Condition I: A = AT and det( A) = 0;
Condition II: A = AT and A has an eigenvalue 1;
Condition III: A = AT .
1
1
(d) What is the least square error solution of Ax = for the unique matrix
1

1
obtained in (c)?
( 98)

(a) N(A) (nullity)

2 0 2 1 1 0 1 1 2 1 0 1 1 2
1 2 0 1 0 2 1 1 2 0 1 1 2 1 4

3 1 4 1 0 1 1 1 2 0 0 1 1 2
1 0 0 1
0 1 0 0
0 0 1 1 2
3 N(A)


http://press.nctu.edu.tw

dim N ( A) = 3 rank A = 4 dim N ( A) = 1


(b) N(A) A N ( A) = C ( AT ) (a)
1
0
N ( A)
1 2

1

(c) rank A = 1 < 4 I det A = 0 I


A II x Ax = x x N ( A)
1 1 0 0
0 0 1 0
Ax = 0 AS = S S =
1 2 0 0 1

1 1 0 1 2
1 0 0 0
0 0 0 0
= A = S S 1 4 4 S
0 0 0 0

0 0 0 0
rank A = 1 (b)
1
0
A v = A = AT A
1 2

1
1 0 1 2 1
0 0 0 0
A = kvvT = k Av = v
1 2 0 1 4 1 2

1 0 1 2 1
9k 4
Av = k ( vT v ) v = v = v k =
4 9
4 0 2 4

1 0 0 0 0
A= III A
9 2 0 1 2

4 0 2 4
1 1
1
T 1
(d) Ax = A Ax = A T

1 1

1 1
rank A A = rank A = 1 A A
T T

(the least square


error solution) AT A


http://press.nctu.edu.tw

4 1
0 1
1 4 6 3
x = x = x =
9 2 1 9 9 2

4 1
3 2
0
x =
0

0

29. Let S = {(1, 0,1),( 2, 2, 2)} in R 3, and W = span(S).

(a) Find an orthonormal basis of W and W .


(b) If x = ( 2, 0, 0), find the closet vector u on W to x.
(c) What is the closest distant from W to x? Please also specify the
corresponding vector z.
(d) Please plot a schematic diagram that specifies the relation of x, u, z, W and
W .
( A98)

1 2
(a) S A = 0 2 W A W AT
1 2

1 2
Gram-Schmidt a1 = 0 a 2 = 2

1 2

1
v1 = a1 = 0
1

2 1 0
aT2 v1 4
v 2 = a 2 T v1 = 2 0 = 2 W
v1 v1 2
2 1 0


http://press.nctu.edu.tw

1 0
1
0 1 W = N ( AT )
2
1 0

1
1 0 1 1 0 1
A =
T
N ( A ) 0
T

2 2 2 0 1 0 1

1
1
0
2
1

(b) closest closet AT Ax = AT b


u = Ax (a)

1 2 0

( )
1
Q= 0 1 u = Q QT Q QT b QT Q = I
1 2 0

1
u = QQ b = 0
T

(c) distance distant z

1
u x z = x u = 0 z = 2
1

(d) W W u W z W
x = u + z


http://press.nctu.edu.tw

30. Consider an n-dimensional vector space over real field R. Often, we use the

standard basis {ei , i = 1,..., n} to span the vector space, where ei is an

n-element vector with the ith element being 1 and all the other elements being

zero. Now given another two bases {vi , i = 1,..., n} and {w i , i = 1,..., n} for

the vector space, where vi is not orthogonal to v j , w i is not orthogonal to

w j , but vi is orthogonal to w j , for i j. Given a vector

x = [ x1 x2  xn ]
T
in the space represented by the standard basis, explain

clearly how you solve the weights {ai , bi ; i = 1,..., n} in an efficient way

(without using matrix inversion or Gauss elimination operations) so that x can be


represented as x = a1 v1 +  + an v n = b1w1 +  + bn w n .
( 96)

v i w j (i j )

x = a1v1 +  + an v n w j

xT w j
xT w j = a1 v1T w j +  + an vTn w j = a j vTj w j vTj w j 0 a j =
vTj w j

xT v i
bi =
wTi vi

a1
[ v1  v n ] = x
an


http://press.nctu.edu.tw

w1T a1 w1 v1
T
0 a1 w1T w1T x

[ v1  v n ] =
= x =
w Tn an 0 w Tn v n an w Tn w Tn x

xT w j
aj = T
w Ti v i = 1 i = 1, 2, , n
v wj
j

w1T

v i w j (i j ) = [ v1  v n ]
1

w Tn

31. Suppose the matrix A has rank r and A = PR, where P is an invertible matrix
and R is the reduced row echelon form of A. Let Col A be the column space of A,

( Row A) be the orthogonal complement of the row space of A with respect to

the dot product, and Null R be the nullspace of R.


(a) Prove that the first r columns of P form a basis of Col A.

(b) Prove that ( Row A ) = Null R.

( 95)

(a) R

I F
R = r F
0 0

A
A A m n P m m
R m n m r

I F I F
A = PR = [p1  p r  p m ] r = Pr P m r r
0 0 0 0
= [ Pr I r Pr F ] = [ Pr Pr F ]

A Pr P r Pr F
r P
A P r Pr r


http://press.nctu.edu.tw

A
(b)

x ( Row A ) x ( Row A ) Ax = 0 (x A 0)

A = PR PRx = 0 P 1 Rx = 0

x N ( R ) ( Null R) ( Row A ) Null R

x N ( R ) Rx = 0 P PRx = Ax = 0

x ( Row A ) Null R ( Row A )


( Row A) = Null R


http://press.nctu.edu.tw

Chapter 5
Determinants


http://press.nctu.edu.tw

1. Which of the followings are correct?

2 1 3 4 0
1 5 2 0 0

(a) The determinant of 1 0 0 3 0 is 130.

2 0 0 1 0
0 0 0 0 2

2 1 3 4 0
1 5 2 0 0

(b) The determinant of 1 0 0 3 0 is 140.

2 0 0 1 0
0 0 0 0 2

2 1 3 4
1 5 2 0
(c) The determinant of is 65.
1 0 0 3

2 0 0 1
2 1 3 4
1 5 2 0
(d) The determinant of is 70.
1 0 0 3

2 0 0 1
2 1 3 4
1 5 2 0
(e) The determinant of is 80.
1 0 0 3

2 0 0 1
( 95)


4 4 2
(cofactor expansion)
2 1 3 4 2 1 3 4
2 1 3
1 5 2 0 1 5 2 0 1 3
= = ( 5 ) 1 5 2 = ( 5 ) 1 = (5) 1 (13) = 65
1 0 0 3 1 0 0 3 5 2
1 0 0
2 0 0 1 0 0 0 5
5 5


http://press.nctu.edu.tw

2 1 3 4 0
2 1 3 4
1 5 2 0 0
1 5 2 0
1 0 0 3 0 = 2 = 2 65 = 130
1 0 0 3
2 0 0 1 0
2 0 0 1
0 0 0 0 2

(a)(c)

2. Let det( A) be the determinant of the matrix A R nn . Which of the followings


are correct?
(a) det( A) 0 A invertible
(b) det( AT ) = det( A)
(c) det( AB ) = det( A) det( B )
(d) If A is invertible, then det( A1 ) = det( A)1.
(e) det(cA) = c n 1 det( A)
( 95)

(a)(b)(c)(d)(e)
det(cA) = c n det( A)

3 0
det(3I ) = = 9 = 32 det( I )
0 3

A n n det(kA) = k n det( A) det(kA) = k det( A) det(i)


http://ccjou.twbbs.org/blog/?p=1926

3. det(2 I n ) = ?
( 97)

det(2 I n ) = 2n det( I n ) = 2n


http://press.nctu.edu.tw

0 a b c d
a 0 e f g

4. Find the determinant of a real matrix K, where K = b e 0 h i .

c f h 0 j
d g i j 0

( 95)

K T = K
(skew-symmetric)

det ( K T ) = det( K ) det( K ) = (1)5 det( K ) = det( K )

det( K ) = det( K ) det( K ) = 0

0 1 2 3
2 6 6 3
5. Let A = .
1 0 0 3

1 2 2 3
(a) By applying row operations to find a lower triangular matrix L with 1s on
its diagonal and an upper triangular matrix U so that A = LU .
(b) Find the determinant of A.
( 95)

(a)
0 1 0 0 1 0 0 0
0 0 1
0 1 2 1 0 0
PA = LU P = L =
0 0 0 1 1 2 1 3 1 0

1 0 0 0 0 1 3 1 4 1
2 6 6 3
0 3 3 9 2
U =
0 0 4 3

0 0 0 9 4
(b) (a) PA = LU det P det A = det L det U
det P = 1


http://press.nctu.edu.tw

9
det L = 1 det U = 2 3 (4) = 54 det A = 54
4

a b c 4a + 5d + 6 g 4b + 5e + 6h 4c + 5 f + 6i
6. Let A = d e f and B = 2a + 3d
2b + 3e 2c + 3 f . If
g h i a b c

the determinant of A is 5, what is the determinant of B?


( 97)

B A det( B )

a b c
B = 2a + 3d 2b + 3e 2c + 3 f
4a + 5d + 6 g 4b + 5e + 6h 4c + 5 f + 6i

()

a b c
B = 3d 3e 3 f 3
5d + 6 g 5e + 6h 5 f + 6i

a b c
B = 3 d e f 6
5d + 6 g 5e + 6h 5 f + 6i

a b c a b c
B = 3 d e f = 3 6 d e f = 3 6 5 = 90
6 g 6h 6i g h i

2I B
7. If A, B R 33 , det( A) = 2, and det( B) = 1, then det 3 =?
0 AB
( 98)


http://press.nctu.edu.tw


2 I3 B
= det(2 I 3 ) det( AB) det(2 I 3 ) = 23 det( I ) = 23
0 AB
2 I3 B
det( AB) = (det A)(det B ) = 2 = 16
0 AB

8. Consider an n n matrix An with the (i,j)-entry being min ( i, j ) , then

(a) det ( A5 ) = 1.

(b) det ( A10 ) = 1.

(c) n > 1, det ( An ) = n.

(d) n > 1, det ( An ) = det ( An 1 ) .

(e) n > 1, An is diagonalizable.


( 95)

An

1 1 1 1 1
1 2 2 2 2

A5 = 1 2 3 3 3

1 2 3 4 4
1 2 3 4 5

1 2
A1 = 1 A2 = 2 A1 1 A1 = 1 A3 = 3 A2 2 A2 + 1 = 1
1 2

1 1 1 1 2 2
A4 = 4 A3 3 A3 + 2 1 2 3 1 1 2 3 = 1
1 2 3 1 2 3


http://press.nctu.edu.tw

An = n An 1 (n 1) An 1 = An 1 = 1

An
(a)(d)(e)

9. Let A be a 5 3 matrix with orthonormal columns, then

(a) det ( AT A ) = 0.

(b) det ( AT A ) = 1.

(c) det ( AAT ) = 1.

(d) det ( AAT ) = 0.

(e) AT has a nontrivial nullspace of dimension 2.


( 95)

5 3 A A = [a1 a 2 a3 ] 5

1 i = j
aTi a j =
0 i j

a1T 1 0 0

AT A = aT2 [a1 a 2 a3 ] = 0 1 0 = I det ( AT A ) = 1
aT3 0 0 1

(column-row expansion)

a1T
3
AAT = [a1 a 2 a3 ] aT2 = ai aTi 5 5 AAT
aT3 i =1

A = [a1 a2 a3 ] 3 rank( AAT ) = rank( A) = 3 < 5

AAT det ( AAT ) = 0


http://press.nctu.edu.tw

rank ( AT ) = rank ( A ) = 3

dim N ( AT ) = 5 rank ( AT ) = 2

(b)(d)(e)

10. Consider the following system of three linear equations in three unknowns:
x1 + x2 + ax3 = 1
x1 + ax2 + x3 = 3
ax1 + x2 + x3 = 2a
where a R.
(a) Find condition on a such that the system has a unique solution.
(b) Find condition on a such that the system has no solution. Find also condition
on a such that the system has many solutions.
(c) Under the condition obtained in (a), use Cramers rule to solve the system.
( 97)

(a)

1 1 a 1 1 1 a 1 1 1 a 1
1 a 1 3 0 a 1 1 a 2 0 a 1 1 a 2

a 1 1 2a 0 1 a 1 a a 0 0 2 a a 2 a + 2
2

2 a a 2 = (1 a)( a + 2) a 1 a 2
3

(b) a = 1 [ 0 0 0 3] a = 2


(c) a 1 a 2 Cramer
1 1 a 1 1 a
3 a 1 2 a 1 1 a
2a 1 1 a + 2 0 (1 a )(a + 2) 2(a 1)2 (a + 2)
x1 = = = = 2
1 1 a 1 1 a (a 1)2 (a + 2)
1 a 1 0 a 1 1 a
a 1 1 0 0 (1 a )(a + 2)


http://press.nctu.edu.tw

1 1 a 1 1 a
1 3 1 0 2 1 a
a 2a 1 0 a 1 a2 ( a 1)( a + 2 ) 1
x2 = = = =
( a 1) ( a + 2 ) ( a 1) ( a + 2 ) ( a 1) ( a + 2 ) a 1
2 2 2

1 1 1 1 1 1
1 a 3 0 a 1 2

x3 =
a 1 2a
=
0 1 a a ( a 1)( a + 2 ) = 1
=
( a 1) ( a + 2 ) ( a 1) ( a + 2 ) ( a 1) ( a + 2 ) a 1
2 2 2

http://ccjou.twbbs.org/blog/?p=4154

11. Solve the following system of linear equations using (a) inverse matrix method
(i.e., Ax = b x = A1b ), and (b) Cramers rule.
3 x1 + x2 + x3 + 2 x4 = 1
x1 + 3 x2 + 2 x3 + 3x4 = 4
2 x1 + x2 + 3 x3 + 3x4 = 3
x1 + 2 x2 + x3 + 2 x4 = 2
( 95)

(a) Gaussian

3 1 1 2 1 1 3 2 3 4 1 3 2 3 4
1 3 2 3 4 3 1 1 2 1 0 8 5 7 11

2 1 3 3 3 2 1 3 3 3 0 5 1 3 5

1 2 1 2 2 1 2 1 2 2 0 1 1 1 2
1 3 2 3 4 1 3 2 3 4 1 3 2 3 4
0 1 1 1 2 0 1 1 1 2 0 1 1 1 2

0 5 1 3 5 0 0 4 2 5 0 0 4 2 5

0 8 5 7 11 0 0 3 1 5 0 0 0 1 2 5 4


http://press.nctu.edu.tw

x1 + 3 x2 + 2 x3 + 3x4 = 4
x2 x3 x4 = 2
4 x3 + 2 x4 = 5
5
x4 =
2
1 5 5
x1 = , x2 = 2, x3 = , x4 =
2 2 2
(b) Cramer
Cramer 5 4 4

(a)



2

3 1 1 2 1 3 2 3
1 3 2 3 0 1 1 1 1
= = 1 (1) 4 ( ) = 2
2 1 3 3 0 0 4 2 2
1 2 1 2 0 0 0 1 2
x1

1 1 1 2
4 3 2 3
3 1 3 3
2 2 1 2
x1 =
3 1 1 2
1 3 2 3
2 1 3 3
1 2 1 2


http://press.nctu.edu.tw

1 1 1 2 4 3 2 3
4 3 2 3 2 1 1 1
=
3 1 3 3 5 0 4 2
2 2 1 2 5 4 0 0 1 2
1 1 1 3 2 3 3 2 3 3 2 3
5
=4 0 4 2 +2 0 4 2 + 5 1 1 1 1 1 1
4
0 0 1 2 0 0 1 2 0 0 1 2 0 4 2
1 5
= 4 2 + 2 (6) + 5 (2) = 1
2 4
3

3 1 1 2 1 4 2 3
1 4 2 3 0 2 1 1
2 3 3 3 0 5 4 2
1 2 1 2 0 5 4 0 1 2 4
x2 = = = = 2
3 1 1 2 1 3 2 2 2
1 3 2 3 0 1 1 1
2 1 3 3 0 0 4 2
1 2 1 2 0 0 0 1 2

3 1 1 2 1 3 4 2
1 3 4 3 0 1 2 1
2 1 3 3 0 0 5 2
1 2 2 2 0 0 5 4 1 2 5
x3 = = =
3 1 1 2 1 3 2 2 2
1 3 2 3 0 1 1 1
2 1 3 3 0 0 4 2
1 2 1 2 0 0 0 1 2

3 1 1 1 1 3 2 4
1 3 2 4 0 1 1 2
2 1 3 3 0 0 4 5
1 2 1 2 0 0 0 54 5
x4 = = =
3 1 1 2 1 3 2 2 2
1 3 2 3 0 1 1 1
2 1 3 3 0 0 4 2
1 2 1 2 0 0 0 1 2


http://press.nctu.edu.tw

12. Answer the following questions.

(a) Let matrix A = [a1 a 2 a3 ] , where a1 , a 2 , a3 are vectors in R 3 . If

4a1 3a 2 + 2a3 = 0, find det( A).

2 1 0
(b) Given the following adjoint of matrix A, adj ( A ) = 4 3 2 , find
2 1 2

det ( A ) , A, and det ( 3 A1 AT ) .

( 96)

(a) 4a1 3a 2 + 2a 3 = 0 A

4 4 0 4
3 a a
a3 ] 3 = 0 3 N ( A)
[1 2
2 2 0 2

Ax = 0 A det( A) = 0

(b) (classical adjointadjugate) adj ( A )

A A adj ( A) = det( A) I

2 2
1
a b 1 d b 1
A1 = = = adj( A)
c d det( A) c a det( A)

A adj ( A ) = det( A) I A 3

det( A) det ( adj( A) ) = det ( det( A) I ) = det 3 ( A) det( I ) = det 3 ( A)

det 2 ( A) = det ( adj ( A ) )

2 1 0
3 2 4 2
det ( adj( A) ) = 4 3 2 =2 (1) = 16 12 = 4
1 2 2 2
2 1 2

det( A) = 2 ()


http://press.nctu.edu.tw

det ( 3 A1 AT ) = 33 det ( A1 ) det( AT ) = 33 det 1 ( A) det( A) = 27

2 1 0 1 1 2 0
1
4 3 2 = 2 3 2 1

1
1
A = adj( A) =
det( A) 2
2 1 2 1 1 2 1

4 1 1
Gauss-Jordan A = 6 2 2
1 0 1

AB n
1. adj( I ) = I
2. adj( AB ) = adj( B )adj( A)

3. ( )
adj AT = adjT ( A)

4. det ( adj( A) ) = det n 1 ( A)

5. adj(kA) = k n 1adj( A)
1
A adj ( A1 ) = ( adj( A) ) =
1
6. A
det A

7. A adj ( adj( A) ) = det n 2 ( A) A


1 A
adj ( adj( A) ) = det ( adj( A) )( adj( A) ) = det n1 ( A) = det n 2 ( A) A
det( A)

A adj ( A) = det( A) I

http://ccjou.twbbs.org/blog/?p=2753


http://press.nctu.edu.tw

13. The set {rank(the adjoint of A) | A R }


77
contains how many integers?

( 97)

adj ( A ) ji (cofactor) Aij = ( 1) det ( M ij )


i+ j

M ij i j (n 1)

A adj ( A) = det( A) I adj ( A) rank( A) A

1
A1 = adj ( A ) adj ( A)
det( A)
1 1
( adj( A) ) = A
det A

A rank ( adj ( A ) ) = n

A det( A) = 0 A adj ( A) = 0 A

rank ( adj ( A) )




adj( AB ) = adj( B )adj( A)
E A = EA

adj ( A ) = adj( A)adj( E ) E adj( E )

rank ( adj ( A ) ) = rank ( adj( A) )

A
rank( A) = r < n R A

I F
R = r r rank( A) = r = n 1
0 0


http://press.nctu.edu.tw

0 0  0
0 0 0
adj( R) =



0 0  1

rank ( adj( A) ) = rank ( adj( R) ) = 1

0 0  0
0 0 0
rank( A) = r < n 1 adj( R) =



0 0  0

rank ( adj( A) ) = rank ( adj( R ) ) = 0

n = 7 {0,1, 7}

http://ccjou.twbbs.org/blog/?p=3056

14. If A is an n n matrix,
(a) Please find the required multiplications by cofactor expansion along the 1st
row to calculate its determinant.
(b) How many multiplications do we need to calculate if we apply the
elementary row operations in calculating the determinant?
(c) From (a) and (b), if you need to write a numerical program to calculate a
matrixs determinant, which method do you prefer? Please justify your
answer.
( 95)

(a) (cofactor expansion)


i+ j
det A = a11C11 + a12C12 +  + a1nC1n Cij = ( 1) det M ij M ij A

i j ( n 1) f (n ) n

det A n a1 j C1 j n ( n 1)

Cij f ( n) = n + nf ( n 1)


http://press.nctu.edu.tw


f (n) = n + nf (n 1) = n + n( n 1 + (n 1) f ( n 2))
= n + n(n 1) + n( n 1) f (n 2)
f (n) = n + n( n 1) + n( n 1)( n 2) +  + n !

1 1
f (n) = 1 + +  + n !
2! (n 1)!

(b) det A = ( 1) d1d 2  d n


k

k d1d 2  d n

n ( n 1)
n(n 1) = n 2 n
2n3 + 3n 2 + n n 2 + n n3 n
n 2 + (n 1) 2 +  + 1 [ n + (n 1) +  + 1] =
=
6 2 3
(c) (b) 10! = 3, 628,800
103 = 1, 000


http://press.nctu.edu.tw

Chapter 6
Eigenanalysis


http://press.nctu.edu.tw

4 2 3
1. Let the matrix A = 1 1 3 . An eigenvalue of the matrix A is 3. Find a
2 4 9

basis for the corresponding eigenspace.


( 95)

1 2 3
A I A 3I = 1 2 3
2 4 6

1 2 3 2 3
0 0 0 N = 1 0

0 0 0 0 1

2 3

1 , 0
0 1


http://press.nctu.edu.tw

2. Answer the following questions.


(a) Please determine all the eigenvalues of the following 3 3 matrix A.

2 1 0
A = 1 1 1
0 1 2

(b) Please determine all the eigenvectors of the above matrix A.


(c) Please determine a matrix U which can diagonalize A according to
U T AU = D. Here D is a diagonal matrix with the eigenvalues of A as its
diagonal elements and U T is the transpose of U.
(d) Please derive the expression for the vector solution x of the following linear

2
equation: ( A I ) x = 2 .
0

( 97)


(a)

2 1 0
A I = 1 1 1 = ( 2)( 3)
0 1 2

023

(b) ( A I ) ( A I ) x = 0

2 1 0 0 1

1 = 0 1 1 1 x1 = 0 x1 =
1
2
6
0 1 2 0 1

0 1 0 0 1

2 = 2 1 1 1 x 2 = 0 x 2 =
1
0
2
0 1 0 0 1


http://press.nctu.edu.tw

1 1 0 0 1

3 = 3 1 2 1 x3 = 0 x3 =
1
1
3
0 1 1 0 1

(c) A

U = [ x1 x 2 x 3 ] U 1 = U T U T U = I

A U T AU = D
1 6 1 2 1 3 0

U = 2 6 0 1 3 D = 2


1 6 1 2 1 3 3

(d) A

2 1 0 2

1 1 1 x = 2 = b

0 1 2 0

(c) A
A = UDU T A I = UDU T U ( I )U T = U ( D I )U T
( A I ) x = b U ( D I )U T x = b
( D I )
U

( D I ) U T ( D I )1 U
x = U ( D I ) 1U T b

1

1 6 1 2 1 3
1 6 2 6 1 6 2

0 1 2 2
1
x = 2 6 0 1 3 1 2
2 0
1 6 1 2 1 3
1 1 3 1 3 1 3

3


http://press.nctu.edu.tw

1 1
+ 2

2
x =


1 1
2

3. Find the eigenvalues and corresponding normalized eigenvector (norm equals to

1 0 0
1) for the matrix A = 2 4 0 . What are those for the transpose matrix AT ?
6 4 2

( 97)


124

3 0 0
1 1
2 0 1
113 5
10 1 2

1 2 2
1
0 2
1
3
3 13
0 1 0


http://press.nctu.edu.tw

4. Find an invertible matrix P that can diagonalize the following matrix A by


P 1 AP.

1 1 1
A = 0 1 0
1 0 1

( 98)

= P 1 AP P

1 1 1
0 1
3
( 2
)
0 = (1 ) + (1 ) = (1 ) (1 ) + 1 = 0 1 = 1
1 0 1

0 1 1
2 = 1 + i 3 = 1 i x1 = 1 x 2 = 0 x3 = 0

1 i i

0 1 1
P = [ x1 x2 x3 ] = 1 0 0
1 i i


http://press.nctu.edu.tw

5. Students A and B were asked to solve the eigenvalues of the same

a b c
matri M = 0 d 1 . Unfortunately Student A mistook the value of d and
0 2 e

obtained the eigenvalues 0, 1, and 3. Student B mistook the value of e and


obtained the eigenvalues 1, 1, and 2. Assume there were no other mistakes
happened when they were solving the eigenvalues.
(a) Find the value of a.
(b) Assume that the sum of correct eigenvalues of M is 1. Find the correct
values of d and e.
(c) Find the correct eigenvalues of M.
( 97)

(a) A d d

a b c
0 d ( )
1 = (a ) (d )(e ) 2 a A
0 2 e

B e e

a b c
0 d 1 = (a ) ( (d )(e ) 2 ) B a
0 2 e

a {0,1,3} {1,1, 2} a = 1
(b) (trace) a + d + e = 1 d + e = 0

A 03 ( d )(e ) 2 = 2 ( d + e) + de
2

d + e = 3 de
2 = 0 e = 1 e = 2 B 1

2 (d )(e ) 2 = 2 (d + e) + de 2
d + e = 1 de 2 = 2 d = 0 d = 1
d + e = 0 d = 1 e = 1

(c) ade M (1 ) ( 2 3)


http://press.nctu.edu.tw

1 3 3

x 2 x2
6. Define the linear operator T on R 2 by T 1 = . Which
x2 3 x1 + x2

statements in the following are correct?


(a) 3 is an eigenvalue of T.
(b) 4 is an eigenvalue of T.
(c) 2 is an eigenvalue of T.

2
(d) is a basis for the eigenspace of T.
3

3
(e) is a basis for the eigenspace of T.
3
( 97)

x x 0 2 x1
T 1 = A 1 =
x2 x2 3 1 x2

1
A I = ( + 2 )( 3) 23
1

2
3



(a)(c)(d)(e)


http://press.nctu.edu.tw

7. Let A be the 3 3 matrix defined below.


(a) Find the eigenvalues of A.
(b) Find an orthonormal basis for R 3 consisting of eigenvectors of A.

4 2 2
A = 2 4 2
2 2 4

( 98)

4 2 2 1 1
(a) 2 4 2 1 = 8 1 A 1 = 8

2 2 4 1 1

1
x1 = 1 (trace)
1

det A = 123 = 32 trace A = 1 + 2 + 3 = 12 2 = 2 3 = 2

1 1
x 2 = 1 x3 = 1

0 2

1 1 1
1 1 1
(b) (a) 1 1 1
3 2 6
1 0 2


http://press.nctu.edu.tw

2 2 3
8. Pick the correct statements regarding the matrix A = 2 1 6 .
1 2 0

(a) There are three sets of linear independent eigenvectors associating with
three distinct eigenvalues.
(b) The determinant of this matrix is 45.

2
(c) 1 is one of the eigenvectors.

0

(d) 5 is an eigenvalue.
(e) The homogeneous linear system Ax = 0 has no non-trivial solution.
( 96)

(a)
A

2 3
3 3 5 3 1 0
0 1

1
5 2
1

(b) det A = ( 3) ( 3) 5 = 45

2 2 3 1
(c) 1 1 0 2

0 0 1 1

2 2 3 2 2
2 1 6 1 = 5
1 2 0 0 4


http://press.nctu.edu.tw

(d) (a)
(e) 45 A Ax = 0 x = 0
(b)(d)(e)

9. Describe all possible 2 2 matrices whose eigenvalues are 0 and 1.


( 96)

( 2 2 )
() 2 2 2
0 1 1 0
a b
2 2 A
c d

a b 1 0 1 d b 1 ad ab
A= =
c d 0 0 ad bc c a ad bc cd bc

abcd ad bc

a 1
10. Consider the 2 2 matrix A = . Find the value a0 for which A has
0 1

repeated real eigenvalues. What happens to the eigenvectors of this matrix as a


approaches a0 ?
( 96)

a 1
A I = = ( a )(1 ) 1 a
0 1

a = 1 a 1 1 a

1 1
1 a 0 a 1


http://press.nctu.edu.tw

a b
11. Let A = be a matrix whose elements are non-negative and satisfy
c d

b 1
a + c = 1 = b + d . Also let P = . Prove that if A I then
c 1

(a) P is nonsingular and calculate P 1 AP,

1 b b 0 1
(b) An as n , if A .
b + c c c 1 0

( 97)

a b b 1 ab + bc a b
(a) P 1 AP AP = =
c d c 1 bc + cd c d

b a b
a + c = 1 = b + d AP =
c c d

b a b
c d = b a AP = P
c b a

P A 1 ( a b ) P

det P 0 a, b, c, d 0 a + c = 1 = b + d
1 0
a, b, c, d 1 A a, d 1
0 1
b, c 0 det P = (b + c) 0 P

1 0
P A P 1 AP =
0 a b

(b)
b 1 1
1
0 b 1 1
A = P P =
n n
n
c 1 0 ( a b ) c 1
b 1 1 0 1 1 1
=
( a b ) b + c c b
n
c 1 0


http://press.nctu.edu.tw

0 1
(a)0 a < 1 0 < b 1 A a b < 1
1 0

b 1 1 0 1 1 1 1 b b
lim An = =
n
c 1 0 0 b + c c b b + c c c

http://ccjou.twbbs.org/blog/?p=2759

12. Let L be the linear transformation of the reflection about the line ax + by = 0,
from R 2 to R 2 , where a 2 + b 2 0.
(a) Find the matrix representation of L with respect to the standard basis.
(b) Find the dimension of the kernel space.
(c) Find a basis of the range space.
( 96)

a b
(a) ax + by = 0
b a

a
b ()

b b a a
L = x L =
a a b b

(1 1 ) A
L
1
b a 1 0 b a b a 1 0 1 b a
A= = 2 a
a b 0 1 a b a b 0 1 a + b
2
b
1 b 2 a 2 2ab
= 2 2
a + b 2ab a b 2
2

(b) A (
) nullity(L)=dim N(A)=0
(c) A (b) A


http://press.nctu.edu.tw

13. Let T be a linear transformation from R 2 (the 2-D x-y coordinates) to R 2


given by rotating counterclockwise around the origin by an angle of = / 4,
followed by the reflection in the y-axis.
(a) Find the matrix representation M of T with respect to the standard basis for
R 2.

(b) T has two eigenvectors q1 = [1 1]T and q 2 = [1 1]T Find their

eigenvalues 1 and 2 .
(c) Find the matrix for T with respect to the q1 and q 2 in (b).
( 97)

(a)

1 0
M T T
0 1

1 0
M M = T T
0 1

1 1 1 1 1
45 y
0 2 1 2 1

0 1 1
45 y
1 2 1

1 1 1 1 1
1 M =
2 2 1 1

(b) T q1 = [1 1]T q 2 = [1 1]
T

1 1 1 1 0 1
Mq1 = 1 1 1 = q1 = 1
2 2
1 = 1 2 = 1

1 1
q1 = , q2 =
1 + 2 1 2


http://press.nctu.edu.tw

(c) (b)
1 0
= M = S S 1 S 1MS =
0 1
1 1
S =
1 + 2 1 2

14. In R 3 , let g be a line through the origin and E be a plane through the origin
such that g is not in E. Determine (geometrically) the eigenvalues and
eigenspaces of the following linear maps:
(a) Reflection in the plane E.
(b) Reflection in the origin.
(c) Parallel projection in the direction of g onto E.
(d) Rotation about g through / 3 followed by rescaling in the direction of g
with factor 6.
(e) Which of these maps admit a basis of eigenvectors?
( 97)

(a) E dim E = 2
11 E E
dim E = 1 1
E
(b) dim R 3 = 3 111
R 3
(c) g ( R 3 ) E

g E 0 g g E

g E
(d) g dim g = 1 6

g cos i sin
3 3

(e)
R 3 (a),(b)


http://press.nctu.edu.tw

15. Let A be an n n matrix that is similar to a lower triangular matrix and has the
distinct eigenvalues 1 , 2 , , k with corresponding multiplicities
m1 , m2 , , mk . What are tr( A) and det( A) ?
( 95)

tr( A) tr( A) = m11 + m22 +  + mk k

det( A) det( A) = 1m1 2m2  kmk

A
() A

16. Find det( A) given that A has p ( ) as its characteristic polynomial.


(a) p ( ) = 3 2 2 + + 5
(b) p ( ) = 4 3 + 7
( 95)

A

3 4

det( A)
det( A I ) = 0 det( A)

(a) det A = p (0) = 5
(b) det A = p (0) = 7


http://press.nctu.edu.tw

17. Which statements are correct?


(a) Assume A is an m n matrix and B is an m p matrix. If X is an n p
unknown matrix, then the system AT AX = AT B always has a solution.
(Here we assume m n. )
7 24
25 i i
25
(b) The matrix possesses a complete orthonormal set of
24 i 7
i
25 25
eigenvectors.

2 0 2
(c) 0 3 0 is not defective.

0 0 2

( 98)

(a) p
A rank A = n
m n

(b) Q = [q1 q 2 ] q1q1 = 1

q2q 2 = 1 q1q 2 = 0

(c) Defective 2 2

0 0 2 1

0 1 0 x = 0 x = 0
0 0 0 0

1
(b)


http://press.nctu.edu.tw

18. Which statements are correct?

2aT2
T
(a) Assume A33 = a1 a 2
a3 and B33 = a1 + aT2 + aT3 . If detA = 2,
aT3

then det(AB 1 ) = 1.

(b) If P33 is a projection matrix that projects any vector in R 3 onto the

1
vector u = 1 , then there must be two eigenvectors that correspond to the
1

eigenvalue of 0.
(c) If A is a 3 3 matrix with 3 distinct eigenvalues 0, 1, 2, then the matrix
( A + I ) must be invertible.
( 98)

0 1 0
(a) B = [ 2a 2 T
a1 + a 2 + a3 a3 ] = A 2 1 0 = AC
0 1 1

det B = det BT = (det A)(det C ) = 2 (2) = 4


1 1
det( AB 1 ) = det A(det B )1 = 2 =
4 2

(b) P x span ( u )


Px = 0 dim span ( u ) = 2

(c) ( A + I ) ( A) + 1 123
det A = 1 2 3 = 6 0
(b)(c)


http://press.nctu.edu.tw

0.6 0.4 0.3


19. Given A = 0.4 0.9 0.2 . What is the sum of all As eigenvalues?
0.3 0.2 0.8

(a) 2.7
(b) 2.1
(c) 2.3
(d) 2.4
(e) 2.5
( 95)

A (trace)
trace( A) = 0.6 + 0.9 + 0.8 = 2.3
(c)

1 3 7 11
0 1 2 3 8
20. Let B = . Find the eigenvalues of B 9 .
0 0 0 4

0 0 0 2
( 95)

B 9 B
1
= 01 2 B9
2
9 01 29 29


http://press.nctu.edu.tw

21. Which statements are correct?


(a) Assume V and W are vector spaces and L : V W is a linear
transformation. Let ker(L) denote the kernel of L and L(S) denote the image
of S for any subspace S of V. If dim(V) = n and dim(W) = m, then
dim(ker(L))+dim(L(V)) = m. (Assume n and m are finite.)
(b) Using the same notations in the previous question, if x ker( L ), then

L ( v + x ) = L ( v ) for any v V .

(c) Let P3 be the space consisting of all polynomial of degree no more than 3,

and D be the differentiation operator on P3. Then, ker( D) = {0}.

(d) If A and B are similar matrices, then det( A I ) = det( B I ) for any
scalar .
( 98)

(a) dim(ker( L )) + dim( L(V )) = n


(b) L( v + x) = L( v ) + L(x) = L( v )

(c) 0 ker( D) = {c c R} ker( D )

1
(d) A B A B

(b)(d)


http://press.nctu.edu.tw

22. If 1 , 2 , 3 , 4 , 5 are all the eigenvalues of the matrix

4 1 0 0 0
1 3 1 0 0

0 1 3 1 0 , then 12 + 22 + 32 + 42 + 52 = ?

0 0 1 3 1
0 0 0 1 3

( 97)

A
A

A2 2 A2 trace ( A2 ) (trace)

A
()

17 7 1 0 0
7 11 6 1 0

A2 = 1 6 11 6 1 trace ( A2 ) = 17 + 11 + 11 + 11 + 10 = 60

0 1 6 11 6
0 0 1 6 10

23. Give an example of a linear operator T from R 2 to itself such that T has no real
eigenvalues, or prove that no such operators exist, i.e., every linear operator
T L( R 2 ) has at least one real eigenvalue.
( 98)

2
0 1
T (x) = x T
2 1 0
i


http://press.nctu.edu.tw

24. Let A = aij be an n n matrix with eigenvalues 1 , , n . Show that


n

(
j =1
j a jj ) = 0.

( 96)

n n n
(
j =1
j a jj ) = j a jj = 0
j =1 j =1
n
trace( A) = a jj
j =1

a11 a12 a1n


a21 a22 a2 n
A I =

an1 an 2 ann

n = 3 ( 3 3 )

a a12 a11 a13 a22 a23


A I = 3 + (a11 + a22 + a33 ) 2 11 + +
a21 a22 a31 a33 a32 a33
a11 a12 a13
+ a21 a22 a23
a31 a32 a33

A 1 n

A I = (1 )(2 ) (n ) n = 3

A I = 3 + ( 1 + 2 + 3 ) 2 ( 12 + 2 3 + 13 ) + 12 3

2 1 + 2 + 3 = a11 + a22 + a33

n 3 n n



http://press.nctu.edu.tw

4 1 0 0 1 cos( + )
1 4 1 0 0 cos(2 + )

25. Let B = 0 1 4 1 0 and x = cos(3 + ) .

0 0 1 4 1 cos(4 + )
1 0 0 1 4 cos(5 + )
b
0

(a) a, b, c R such that Bx ax = 0 , where (a, b, c) = ? Please express

0
c
a, b, and c in terms of and .
(b) All the eigenvalues of B are _________.
( 98)

(a) a
b 4 a 1 0 0 1
0 1 4a 1 0 0

( B aI 5 )x = 0 B aI = 0 1 4a 1 0

0 0 0 1 4a 1
c 1 0 0 1 4 a
23 4 a

cos ( + ) + (4 a) cos ( 2 + ) + cos ( 3 + ) = 0

cos ( + ) + cos ( 3 + ) = 2 cos ( 2 + ) cos

(4 a) cos ( 2 + ) = 2 cos ( 2 + ) cos a = 4 + 2 cos

a b c
b = 2 cos cos ( + ) + cos ( 2 + ) + cos ( 5 + )
= cos ( 2 + ) cos + cos ( 2 + ) + cos ( 5 + )
= cos ( 5 + ) cos

c = cos ( + ) + cos ( 4 + ) 2 cos cos ( 5 + )


= cos ( + ) + cos ( 4 + ) cos ( 6 + ) cos ( 4 + )
= cos ( + ) cos ( 6 + )

(b) b = c = 0 a B b = cos ( 5 + ) cos = 0


http://press.nctu.edu.tw

c = cos ( + ) cos ( 6 + ) = 0 5 = 2 k

2 k 2 k
= a = 4 + 2 cos = 4 + 2 cos k = 0,1, 2,3, 4
5 5
(a) B B

B = 4 I 5 + P + P 4 P 5 5

0 0
1 0 0 0 0 0 0 1
0 0
0 1 0 1 0 0 0 0

P = 0 0 P 4 = 0 1 0
0 0 1 0 0 P

0 0
1 0 0 0 0 1 0 0
1 0
0 0 0 0 0 0 1 0
y Py = y

( ) (
By = 4 I 5 + P + P 4 y = 4 + + 4 y B )
= 4 + + 4 P

1 0 0 0
0 1 0 0
det ( P I ) = 0 0 1 0
0 0 0 1
1 0 0 0
1 0 0 1 0 0 0
0 1 0 1 0 0
= +
0 0 1 0 1 0
0 0 0 0 0 1
= 5 + 1 = 0

2 ik

k = e 5
k = 0,1, 2,3, 4 4 = 1 B
2 ik 2 ik
2 k
k = 4 + e 5
+e 5
= 4 + 2 cos k = 0,1, 2,3, 4
5

http://ccjou.twbbs.org/blog/?p=4044


http://press.nctu.edu.tw

26. A m n matrix A is full rank if rank( A) = min(m, n). Which of the followings
are correct?

(a) A full rank AAT invertible; AAT invertible A full rank

(b) A full rank AAT invertible; AAT invertible A full rank

(c) A full rank AAT invertible; AAT invertible A full rank

(d) A full rank AAT invertible; AAT invertible A full rank


(e) All AAT s eigenvalues are strictly positive.
( 95)

A m n AAT m m
rank( A) = rank( AAT ) = rank( AT A) A
(full rank) rank( A) = min{m, n}
m > n rank( AAT ) = rank( A) = n m AAT
m n rank( AAT ) = rank( A) = m AAT
A AAT

AAT rank( A) = rank( AAT ) = m


rank( A) min{m, n} rank( A) = min{m, n}
(b)

xT AAT x = ( AT x ) AT x = AT x 0 AAT x = x
T 2

2
xT AT Ax = xT x = x 0


(b)

http://ccjou.twbbs.org/blog/?p=3954


http://press.nctu.edu.tw

27. True or false.


(a) If A and B are invertible matrices in M nn ( F ) and B is similar to A, then,
for any integer k > 0, Ak and B k are similar.

(b) Let T : R n R n be linear transformation. If T (x1 ) = T (x 2 ), then x1 = x 2

when nullity(T ) = 0.
(c) If a vector space V is the direct sum of W1 and W2 , then W1 W2 = .
(d) {0} is a linear independent set.

(e) {1, x, x }
2
is an orthonormal basis for P3 ( F ) .

(f) The vectors in an eigenspace of a linear operator T are eigenvectors of T.


( 95)

(a) B A P B = PAP 1

B k = ( PAP 1 ) ( PAP 1 ) = PAk P 1 B k Ak AB




A = IAI 1
(b) nullity(T ) = 0 T ( x) = 0 x = 0 T (x1 ) = T (x 2 )
T (x1 ) T (x 2 ) = T (x1 x 2 ) = 0 x1 x 2 = 0

(c) W1 W2 V
v V v = w1 + w 2
w1 W1 w 2 W2 V W1 W2 direct sum
V = W1 W2 W1 W2

W1 W2 = {0} W1 W2 =

(d) 3 0 = 0 3 0 {0}
{0}

(e) P3 ( F ) {1, x, x 2 } 3

(f) (span)
(a)(b)(f)


http://press.nctu.edu.tw

28. Which of the followings are correct?


(a) If A and B are invertible, then so is AB.
(b) If A is invertible, then so is AT .
(c) Eigenvalues of a triangular matrix are its diagonal elements.
(d) It is impossible to have real-valued matrix A such that A2 = I .
(e) It is possible that AB BA.
( 95)

(a) AB AB
det AB = det A det B 0 AB
(b) (a) A det AT = det A 0
AT
(c)
(d) a 2 = 1 A2 = I

A2 1 1 A

i 0 1
i A A = S S
0 i

a a
S = a, b R
1 bi 1 + bi
1 a
1
S A = 1 + b 2
b 1
a
1 0
A2 = a 0 b 0
0 1
A2 = I
(e)
(a)(b)(c)(e)


http://press.nctu.edu.tw

0 1 30
4
B C * * 0
29. Consider the matrix A = = .
0 D 0 0 61

0 0 16
(a) Indicate the 2 by 2 matrix B if the eigenvalues of B are 1 and 2.
(b) Find the eigenvalues of the 4 by 4 matrix A.
1
1 1 1 0 1 1
(c) Let B =
5
, please find out the unknown entries x,
x y 0 x y
y, and .
( B98)

0 1
(a) B = det ( B I ) = 0 det ( B 2 I ) = 0 a + b = 1
a b
0 1
a + 2b = 4 a = 2 b = 3 B =
2 3
(b) A B D
12 57
1 1
(c) B 5 = 25
x y
1 2 x = 1 y = 2

4 3
30. Compute A8 , where A = .
2 1

( 95)

(power) A = S S 1 Ak = S k S 1

4 3 1
= 2 3 + 2 = ( 1)( 2) 1 = 1 x1 = 2 = 2
2 1 1

1
3 1 3 1 0 1 3
x 2 = A =
2 1 2 0 2 1 2


http://press.nctu.edu.tw

1 3 1 0 2 3 3 28 2 3 3 28 766 765
A =
8
8 = 9 =
1 2 0 2 1 1 2 2 3 29 510 509

100

cos 100 sin
100
31. =?
sin cos

100 100
( 98)


100
100
cos sin 1 0
=
sin cos 0 1

1 2 1 2 0
32. Let A = 1 4 1 4 1 2 .
1 4 1 4 1 2

(a) Find the characteristic polynomial of A.


1
(b) Find a nonsingular matrix P such that P 1 AP = diag 1, 0, .
4
(c) Find A , for n 1.
n

( 96)

1 2 12 0
1 1 4 12 114 12
(a) A I = 14 1 4 1 2 =
2 14 1 2 2 1 4 1 2
14 14 1 2

1
A I = ( 1)
4
1
= 0 1
4


http://press.nctu.edu.tw

1 1 2
(b) P P = 1 1 1 P
1 0 1

1
= 1 0
4
1
1 1 2 1 1 1 2
(c) n 1
A = P P = 1 1 1 0
n 1 1 1

1 0 1 4 n 1 0 1

1 + 2(4 n ) 1 + 2(4 n ) 1 4 n+1


1
An = 1 4 n 1 4 n 1 + 2(4 n )
3
1 4 n 1 4 n 1 + 2(4 n )

1 30
33. Find an upper triangular matrix A that satisfies A3 = .
0 8

( 95)

A3 1 8
1
1 10 1 10 1 0 1 10
A3 = S 3 S 1 =
0 3 0 3 0 8 0 3

1
1 1 10 1 0 1 10 1 10
A = S S = =
0 3 0 2 0 3 0 2


http://press.nctu.edu.tw

2 6
34. Compute cos( A) for A = .
1 3

( 96)

cos (Euler)
ei + e i
ei = cos + i sin cos =
2
eiA + e iA
cos A = eiA e iA
2

3 2
1 = 0, 2 = 1 x1 = , x2 = A
1 1

1
1 3 2 0 0 3 2
A = S S = A
1 1 0 1 1 1
eiA = Sei S 1 e iA = Se i S 1 cos A

ei + e i 1
cos( A) = S S
2

ei + e i 1 e0 0 e 0 0 cos 0 0 3 2
= i
+ i
= S =
2
2 0 e 0 e 0 cos1 1 1

3 2 1 0 1 2 3 2 cos1 6 6 cos1
cos A = =
1 1 0 cos1 1 3 1 + cos1 2 + 3cos1

cos ( 2 A )

cos ( 2 ) = 2 cos 2 1 cos ( 2 A ) = 2 cos 2 A I


http://press.nctu.edu.tw

0 1 0 0
0 0 1 0
35. X = . X 50 = ?
0 0 0 1

1 0 0 0
( 96)


X (1234)(2341)
X
(1234) (2341) (3412) (4123) (1234)
(1234) 4 X X 4 = I
X 50 = X 48 X 2 = X 2 (3412)
0 0 1 0
0 0 0 1
X =
50
1 0 0 0

0 1 0 0

36. Suppose that a 3 3 matrix A has eigenvalues = 0, 1, and 1 with the


corresponding eigenvectors (0,1, 1)T , (0,1,1)T , (1, 1,1)T , respectively.
(a) Find the matrix A.
(b) Compute Ak for an arbitrary positive integer k.
( 97)

1
0 0 1 0 0 0 0 0 1
(a) A A = 1 1 1 0 1 0 1 1 1

1 1 1 0 0 1 1 1 1

1
0 0 1 1 1 2 1 2 1 0 0
1 1 1 = 0 1 2 1 2 A = 1 1 2 1 2

1 1 1 1 0 0 1 1 2 1 2

0 0 1 0 0 0 0 0 1 1

Ak = 1 1 1 0 ( 1) 0 1 1 1
k
(b)
0 1 1 1 1
1 1 1 0


http://press.nctu.edu.tw


1 0 0
k
( 1) ( 1)
k

Ak = 1
2 2

( 1) ( )
k k
1
1
2 2

2 1 1
37. Find an orthogonal matrix C such that the matrix A = 1 2 1 is transformed
1 1 2

into a diagonal matrix by C 1 AC = C T AC. Which property of A guarantees that


you can find such a C and the corresponding diagonal matrix?
( 97)

2 1 1
A = 1 2 1 114 A
1 1 2

1 1
= 1 1 0
0 1

1 1
1 1
Gram-Schmidt 1 1 = 4
2 6
0 2

1 1 2 1 6 1 3
1
1 C = 1 2 1 6 1 3
3
1
0 2 6 1 3

1 0 0
C AC = 0 1 0
T

0 0 4


http://press.nctu.edu.tw

38. An n n matrix A is called diagonalizable if A = PDP 1 for some diagonal


n n matrix D and some invertible n n matrix P. Choose the following
matrix which is diagonalizable.

1 0 0
(a) 4 2 5

4 5 8

0 1
(b) 0 0

3 0 0
(c) 0 1 2

0 2 1

5 5 6
(d) 0 1 0

3 2 4

5 0 0
(e) 0 0 2

0 2 0

( 97)


(1) P (orthogonal
matrix)
(2)
(3)
n

0
(a) 33 1 3 1
1


(b) 00


http://press.nctu.edu.tw

1

0
(c)

1
(d) 2 1 1 1 0
1


(e) 2i 2i 5

(c)

39. Suppose there is an election every year in a country and the total population of
this country remains fixed. If 60% of the people voted for K Party whereas 40%
of the people voted for D Party in the election last time. However, 8% of K Party
voters and 4% of D Party voters change their minds and vote for the rival party
each year. What will the percentage of K Party and D Party voters be after n
years, when n approaches infinity?
( 98)

K t Dt t K D
K t +1 = 0.92 K t + 0.04 Dt
K 0 = 0.6 D0 = 0.4
Dt +1 = 0.08Kt + 0.96 Dt

K 0.92 0.04 K t
ut +1 = t +1 = = Aut A
Dt +1 0.08 0.96 Dt

1 1
1 = 0.88 2 = 1 x1 = x 2 = A
1 2
1
1 1 1 0.88 0 1 1
A = S S =
1 2 0 1 1 2

1
1 1 (0.88)n 0 1 1 0.6
u n = S n S 1u 0 = n
1 2 0 1 1 2 0.4

1
1 1 0 0 1 1 0.6 1 3
un =
1 2 0 1 1 2 0.4 2 3


http://press.nctu.edu.tw

http://ccjou.twbbs.org/blog/?p=2821

40. Let A be a diagonalizable n n matrix with

p (t ) = ( 1) ( t 1 )( t 2 ) ( t n ) = cnt n + cn1t n 1 +  + c0 as its characteristic


n

polynomial, then
(a) 1 , 2 , , n must be distinct.

trace ( A) = k =1 k .
n
(b)

cn 1 = ( 1) trace ( A ) .
n 1
(c)

(d) c0 = det ( A) .

(e) There must exist an n n matrix C such that C 3 = A.


( 95)

2 0
(a)
0 2

22
(b)
(c) p (t ) t n 1
n

= ( 1)
n 1 n 1
cn 1 = ( 1) ( 1 2  n ) = ( 1) trace ( A)
n
i
i =1
n
(d) t = 0 p (t ) p (0) = ( 1) ( 1 )( 2 ) ( n ) = i = c0
n

i =1
n
det ( A) = i c0 = det ( A)
i =1

1
1
(e) A A = S S =


n

11 3
1 3 1 1
C = S S = S
S
13
n


http://press.nctu.edu.tw

C 3 = S ( 1 3 ) S 1 = S S 1 = A
3

(b)(c)(d)(e)

41. Choose one or more TRUE statement(s) from the followings:


(a) If A and B are invertible n n matrices, then ABT is also invertible.
(b) If A and B are n n matrices such that B is invertible, then

det ( BAB 1 ) = det ( A ) .

(c) For any m n matrix A and n p matrix B, the nullspace of B is


contained in the nullspace of AB.
(d) An n n matrix A is diagonalizable if and only if there is a basis for R n
consisting of eigenvectors of A.
(e) Distinct eigenvectors of a symmetric matrix are orthogonal.
( 95)

(a) A B det ( A) 0 det ( B ) 0

det ( ABT ) = det( A) det ( BT ) = det( A) det( B ) 0 ABT

ABT A B

1 0
1 0 0 1 0
AB =
T
0 1 = A B
0 1 0 0 0 0 1

ABT A A B

(b) det ( BAB 1 ) = det ( B ) det ( A) det ( B 1 ) = det( B ) det ( A ) det( B )1 = det( A)

(c) N ( B ) N ( AB) x N ( B ) Bx = 0

ABx = A0 = 0 x N ( AB ) N ( B ) N ( AB)

A = 0
(d) n n A n
R n


http://press.nctu.edu.tw

3 0 0
R A = 0 2 5 3
n

0 1 2

1 3 i
i A

3
A = S i S 1

i

(e)


1 x
Ix = x

(a)(b)(c)
(a)(b)(c)(d)

42. Let T be a linear operator on an n-dimensional vector space V with ordered basis
. We define the characteristic polynomial f (t ) of T to be the characteristic

polynomial of A = [T ] , where [T ] denotes the matrix representation of linear

operator T in the ordered basis . That is, f (t ) = det( A tI n ), where det() is


the determinant of the indicated matrix, and I n is the n-by-n identity matrix.
Prove that this definition of characteristic polynomial of a linear operator is
independent of the choice of ordered basis . That is,

( ) (
det [T ] tI n = det [T ] tI n ) for any ordered bases and of V.

( 97)



T : V V

A = [T ] B = [T ]


http://press.nctu.edu.tw

n n P P

A = [T ] B = [T ]

A = [T ]

P 1 P
B = [T ]

A = PBP 1 A B A tI

A tI = PBP 1 PtIP 1 = P ( B tI ) P 1

( )
det ( A tI ) = det P ( B tI ) P 1 = det P det ( B tI ) det P 1 = det ( B tI )

P
V U n n V U
P = U 1V

Standard Basis
V U
-Basis 
P -Basis

43. Suppose that T is a linear operator on a finite-dimensional inner product space V


over the field of real number with the distinct eigenvalues 1 , 2 , , k . Assume
that T is self-adjoint. For each i, (1 i k ), let Wi be the eigenspace of T
corresponding to the eigenvalues i , and let Ti be the orthogonal projection of
V on Wi . Prove that T = 1T1 + 2T2 +  + k Tk .
( 97)

T (self-adjoint)
Hermitian T = T
(spectral theorem) A
T () A
Q = Q 1 AQ = QT AQ

1 q1
T

A = QQT = [q1  q n ]
= q qT +  + q qT
1 1 1 n n n

T
n q n


http://press.nctu.edu.tw


T V
W T W T-(T-invariant) W T-
x W w W W T- T (w ) = Aw W

x Aw w T Ax = ( w T Ax ) = xT AT w = xT Aw = 0
T

T (x) = Ax W W T-

q1 T T (q1 ) = 1q1 W
T- W T- W W
dim W + dim W = dim V = n dim W = 1 dim W = n 1

n = 1 q1 n > 1 W {q 2 , , q n }

q1 W q1T q j = 0 ( j = 2, , n) {q1 , q 2 , , q n }

Q = [q1  q n ] = Q 1 AQ = QT AQ

qi qTi

44. Answer the following questions.


(a) If a0 = 2, a1 = 3, and an +1 = 3an 2an 1 , for n 1, use generating
function method to find the formula for an .
(b) Redo part (a) using eigenvalue method.
( 96)

(a) an +1 = 3an 2an 1

an = r n r n +1 = 3r n 2r n 1 r n 1

r 2 = 3r 2 1 = 1 1 = 2

an = c11n + c2 2n = c1 + c2 2 n a0 = 2 = c1 + c2

a1 = 3 = c1 + 2c2 c1 = c2 = 1 an = 2n + 1

(b)


http://press.nctu.edu.tw

an = an
an +1 = 2an 1 + 3an


n
an 0 1 an1 0 1 a0
a = 2 3 a = 2 3 a
n +1 n 1

n
0 1
12
2 3

1 1

1 2

n 1
0 1 1 1 1 0 1 1 2 2n 2n 1
=
2 3 1 2 0 2n 1 2 = n +1
2 2 2n +1 1

an = ( 2 n 1) a1 ( 2n 2 ) a0 an = 2n + 1

45. Assuming that A is a non-definite matrix with distinct eigenvalues, one can

verify that both u1 (t ) = Xe t c and u 2 (t ) = i =1 ci ei t xi are valid solutions of


n

du
= Au, where c = [ c1 c2  cn ] ,
T
the differential equation
dt

X = [ x1 x 2  x n ] , and = diag ([ 1 2  n ]) . i and xi ,

i = 1,..., n, are eigenvalues and eigenvectors of A, respectively. Are the solutions


u1 (t ) and u 2 (t ) equivalent to each other? Why or why nor?
( 96)

u1 (t ) = u 2 (t )


http://press.nctu.edu.tw

c1
1t n t
u 2 (t ) = c1e x1 +  + cn e x n = e x1 1t
 e x n
n t

cn
e1t c1

= [ x1  xn ]
t
= Xe c = u1 (t )
ent cn


46. Assume M 22 denotes the vector space consisting of all 2 by 2 matrices. Let
T : M 22 M 22 be a linear transformation defined by

a b a + b b + c a b
T = for any M 22 .
c d c + d d c d

(a) Find the eigenvectors and eigenvalues of this transformation.

1 1 1 1 0 0 0 0
(b) We choose B = , 0 0 , 1 1 , 1 1 as the basis of M 22 .
0 0

a b a b
Both and T can be expressed as a linear combination
c d c d
of these four matrices. Assume we have

a b 1 1 1 1 0 0 0 0
c d = x1 0 0 + x2 0 0 + x3 1 1 + x4 1 1 and

a b 1 1 1 1 0 0 0 0
T = y1 + y2 + y3 + y4 . Please find
c d 0 0 0 0 1 1 1 1

[ y1 y4 ] = A [ x1 x4 ] .
T T
the matrix A such that y2 y3 x2 x3

( 96)

(a) M 22 ( 2 2 ) 4 1


http://press.nctu.edu.tw

a a 1 1 0 0 a
b 0
b 1 1 0 b
T = B =
c c 0 0 1 1 c

d d 0 0 0 1 d

1 1 0 0
0 1 1 0
= (1 ) = 0
4
B I =
0 0 1 1
0 0 0 1
1
0
1 0
1111 2 2
0 0 0

0
a
b
(b) v =
c

d

x1
x
T ( v ) = Bv v B [ v ]B = 2 B
x3

x4

a 1 1 0 0 x1 x1
b 1 1
0 0 x2 x
S = = P 2
c 0 0 1 1 x3 x3

d 0 0 1 1 x4 x4

v = P [ v ]B T ( v) = P [T ( v)]B T ( v ) = Bv

P [T ( v) ]B = BP [ v ]B P 1

[T ( v)]B = P1BP [ v ]B = A[ v ]B A = P 1BP P (block


1 1 0 0
0 0
1 1 1
diagonal) P 1 P 1 =
2 0 0 1 1

0 0 1 1


http://press.nctu.edu.tw

3 1 1 1

1 1 1 1 1 1
A = P BP =
2 0 0 3 1

0 0 1 1

B 0
A n n A =
0 C
B m m C k k n = m + k

B 0 B 1 0 BB 1 0
1
= = I
0 C 0 C 0 CC 1

B 1 0
A1 = 1

0 C
http://ccjou.twbbs.org/blog/?p=2366

47. Let v = [1 2 3 4 5] .
T

(a) Please find det ( I + vvT ) , where det() denotes the determinant of a matrix

and I is the identity matrix.


(b) What is the product of the eigenvalues of the matrix I + vv T ?
( 96)

(a) 5 5
(b)
(b) I + vv T

5 5


I + vv T v

( ) ( )
I + vvT v = Iv + v vT v = v + v v = 1 + v
2
( 2
) v = 56v
I + vv T 56 v


http://press.nctu.edu.tw

4 I + vv T

( I + vv ) = I T + ( vvT ) = I + vvT
T T
T

4 x i (i = 1, 2,3, 4) vT xi = 0

( I + vvT ) xi = xi + v ( vT x i ) = xi + 0 v = xi x i 4

det ( I + vvT ) = 1 1 1 1 56 = 56

(b) (a)

http://ccjou.twbbs.org/blog/?p=2235

n if vT u c
48. If u, v R n and rank( I n + uvT ) = , then (c, d ) = ?
d if v u = c
T

( 97)

u v rank ( I + uvT )


(kick the process)

( I + uvT )

x x

( I + uv ) x = x + ( v x ) u
T T

vT x = 0 vT x 0 vT x = 0 ( I + uvT ) x = x 1

x v v (orthogonal complement)

( n 1) 1 ( n 1) vT x 0


http://press.nctu.edu.tw

( I + uv ) x = x + ( v x ) u x u u
T T

( I + uv ) u = Iu + ( v u ) u = (1 + v u ) u (1 + v u )
T T T T

( n 1) 1 x v

(1 + vT u ) u

n rank ( I + uvT ) = n v T u 1

v T u = 1( I + uvT ) u = 0 0 u

1 dim N ( I + uvT ) = 1 rank ( I + uvT ) = n 1

c = 1 d = n 1

n A i
i = 1, 2, , n xi ( I n + A) i + 1
xi Axi = i xi

( I + A)xi = xi + i xi = (1 + i )xi A = uvT u v

rank-one rank( A) = 1 ()

1 2 3 1
A = uv = 1 [ 2 3 1] = 2 3 1 x v Ax = uvT x = 0
T
2 4 6 2

( n 1) 0 v

u Au = uvT u = ( vT u ) u ( vT u )

( I n + A) ( n 1) 1 (1 + vT u ) 1

N ( vT ) v (1 + vT u )


http://press.nctu.edu.tw

(1) A m n ( I n + AT A) rank( I n + AT A) = n

( xT AT Ax AT A )

(2) u R n u = 1 trace( I n + uuT ) = n + 1

(3) u R n u = 1 k det( I n + kuuT ) = 1 k

( I n + kuuT ) 2 = I n

http://ccjou.twbbs.org/blog/?p=4741

49. Let u be an unit vector in R n and H = I 2uuT . Please answer the following
questions (must have reason or counterexample):
(a) Is H a symmetric and orthogonal matrix?
(b) Is H diagonalizable?
(c) Find H 1 and H 2 .
(d) Please find all the eigenvalues of H.
(e) Find the trace, the rank and the determinant of H.
(f) Find a matrix H, as stated above, such that Hx = e1 , where

x = (1/ 3, 2 / 3, 2 / 3)T and e1 = (1, 0, 0)T .

(g) Is it possible to find a matrix H, as stated above, such that Hx = e1 , where

x = (1, 3, 2)T and e1 = (1, 0, 0)T ?

( 97)

H T = ( I 2uuT ) = I T 2 ( uT ) uT = I 2uuT = H H
T T
(a)

H T = H 1 H T H = I u

uT u = 1 H T H = ( I 2uuT ) = I 4uuT + 4uuT uuT = I


2


http://press.nctu.edu.tw

(b) (a) H

H
(c) (a) H 1 = H T = H H 2 = HH = H T H = I

(d) Hu = ( I 2uuT ) u = u 2u = u H

1 u x i u xi u

Hxi = ( I 2uuT ) xi = xi 2u 0 = xi 1

1 ( n 1) 1
n
(e) (d) trace( H ) = i = (n 1) 1 = n 2
i =1
n
det( H ) = i = 1 (1) n 1 = 1 det( H ) 0 H
i =1

rank( H ) = n
(f) H 3 3 H

1 1
1
Hx = ( I 2uu ) 2 = 0
T

3
2 0

Hx = ( I 2uuT ) x = x 2 ( uT x ) u = e1 2 ( uT x ) u = x e1

u ( x e1 ) u = k ( x e1 )

( )
2k 2 ( x e1 ) x = 1 k =
T 1

(
2 ( x e1 ) x
T
)
1
1 1
u= ( x e1 ) = 1 u = 1
(
2 ( x e1 )
T
x) 3
1

1 0 0 1 1 1 1 2 2
2
H = 0 1 0 1 1 1 = 2
1
1 2
3 3
0 0 1 1 1 1 2 2 1

(g) (f) H Hx = e1 ( x e1 ) x > 0


T

k u = 1 2 ( x e1 ) x = x e1 ( T
) 2


http://press.nctu.edu.tw

e1 = 1 x = 1 x 1

H Hx = b
H

H = I 2uuT u = 1 Householder u

Householder

http://ccjou.twbbs.org/blog/?p=3272

2 0 1 0 2 1 0 0
0 2 0 1
50. Let A = , and B = PAPT = 1 2 0 0 with P
1 0 2 0 0 0 2 1

0 1 0 2 0 0 1 2
being a permutation matrix. Denote the (i,j)-entry of P as Pij , then

(a) P12 P22 = 1.


(b) P31 = 1.
(c) P43 = 0.

(d) trace ( P ) = 2.

(e) PT = P 1.
( 95)

4 4! = 24

(1) P 1 = PT
(2) B = PAPT = PAP 1 A (similar) B
()
(3) A B

5 (e)(a)
1 0 P12 P22 = 0


http://press.nctu.edu.tw

P B = PAPT
A B A = U U T B = V V T U V
A B
= U T AU B = V V T
B = VU T AUV T = (VU T ) A(VU T )T P = VU T
P V = PU
U V

B

2 1 0 0
1 2 0 0 2 1 2 1
B I = =
0 0 2 1 1 2 1 2
0 0 1 2
= 1 133
1 0
0
1 1 1
1 A , B
2 1 2 0

0 1
1 0 1 0
1 0 0
1 1 1 1 1
, 3 A , B
2 0 2 1 2 1 2 0

0 1 0 1
1 0
1
1 1 0
, B
2 0 2 1

0 1
(1 3) UV
UV V = PU
1 0 1 0 1 0 1 0
0 1 0 1
U =
1 V = 1 1 0 1 0
2 1 0 1 0 2 0 1 0 1

0 1 0 1 0 1 0 1
V = PU
1 0 0 0
0 0 1 0
P = VU T = (1324)
0 1 0 0

0 0 0 1


http://press.nctu.edu.tw

B = PAPT

UV
1 0
0 1
1 A 1 + 2
1 0

0 1
1 0
0 1
1 + 2 3 A
1 0

0 1
1 0 1 0
0 1 0 1
1 + 1 +
1 2 0 1 2 0

0 1 0 1

1 1 1 1 a1 b1 c1 d1

2 2 2 a b c1 d1
U = 2 V = 1 1

1 1 1 1 a2 b2 c2 d 2

2 2 2 2 a2 b2 c2 d2

V = PU P
(1???) a1 = 1 b1 = 1 c1 = 1
d1 = 1 (13??)
(132?) a2 = 2 b2 = 2 c2 = 2d 2 = 2 (134?) a2 = 2
b2 = 2 c2 = 2 d 2 = 2 (1324)(1342)
4 2 = 8
(1324)(1342)(2413)(2431)(3124)(3142)(4213)(4231)

p ( p(1,3), p(2, 4) ) p (m, n) mn

nm 24 4
8 AB

8 (b)(c)(d)
(e)


http://press.nctu.edu.tw

0 1 0
51. Given the linear operator T with standard matrix [T ]E = 1 0 1 and
0 0 0

1 9 6
B-matrix [T ]B = 0 7 4 , which can be a correct basis for B?
2 11 8

1 3 4

(a) 2 , 5 , 3
1 4 9

1 3

(b) 4 , 6
2 2

1 0 1

(c) 1 , 2 , 2
2 1 1

1 3 2

(d) 2 , 5 , 3
3 2 4

(e) None of the above.
( 97)

P B P B E
E B

[p1 p2 p3 ] [T ]E
 [T ]E [p1 p2 p3 ] = [T ]E P
P P
1 0 0 1 9 6
[p1 p2 p 3 ]B = 0 1 0 [T ]B 0 7 4


0 0 1 2 11 8

B {p1 , p 2 , p3 } B I


http://press.nctu.edu.tw

[TB ] = P 1 [TE ] P P [T ]B = [T ]E P (

)
(a)(b)
(c)

1 0 1 1 9 6 1 2 2
P [T ]B = 1 2 2 0 7 4 = 3 1 2

2 1 1 2 11 8 0 0 0

0 1 0 1 0 1 1 2 2
[T ]E P = 1 0 1 1 2 2 = 3 1 2

0 0 0 2 1 1 0 0 0

det P = 1 P
(c)

[T ]E [T ]B

[T ]E = U U 1 [T ]B = V V 1 = U 1 [T ]E U

[T ]B = V V 1 = VU 1 [T ]E UV 1 = (UV 1 ) [T ]E (UV 1 ) P = UV 1
1

0 1 0
[T ]E = 1 0 1
0 0 0

1 1 1 1 1 1
= 011 0 1 1 U = 0 1 1

1 0 0 1 0 0

1 9 6 6 5 3
[T ]B = 0 7 4 = 0 1 1 4 4 2

2 11 8 7 6 4


http://press.nctu.edu.tw

6 5 3 2 1 1
V = 4 4 2 V = 1 3 2 0
1
7 6 4 2 1 2 2

3 1 3 3 1 3

P = UV 1
= 3 1 2 B 3 , 1 , 2

2 1 1
2 1 1
(c)

2 1 3
U = 0 1 3
2 0 0

6 5 3 9 1 8

V = 4 4 2 P = UV = 7 0 6
1
7 6 4 4 2 2

P [T ]E [T ]B

P [T ]B = [T ]E P

P [T ]B [T ]E P = 0 P

1 1 9 25
[T ]E = 0
[T ]B =
11
11
1 4
a b
P =
c d
T ( P ) = P [T ]B [T ]E P

a b 10a + 4b c 25a + 10b d


T = 25c + 10d
2 2 4
c d 10c + 4d


http://press.nctu.edu.tw

a 10 4 1 0 a

b 25 10 0 1 b
T =
c 0 0 10 4 c
d 0 0 25 10 d

4 4 T ( P ) = 0

10 4 1 0 1 2 5 0 1 25
25 10 0 1 0 0 1 2 5

0 0 10 4 0 0 0 0

0 0 25 10 0 0 0 0

2 5 1 25 2 5 1 25

1 0 1 0
, P = +
0 2 5 0 2 5

0

1



0

1

2
5 25
2 2 P =
2

5

2

5 25 2
det P = = 0 0
2 25

5

3 3 T ( P )

9 9

http://ccjou.twbbs.org/blog/?p=2399


http://press.nctu.edu.tw

Chapter 7
Quadratic forms


http://press.nctu.edu.tw

1. Given an m by n matrix A = [a1 a2  an ] , where ai ia j = 0, i j ,

1 i , j n.
(a) What can you say regarding the properties of AT A ? (at least 3 statements
to be made for full score)
(b) Let m = n = 3 and f (x) = Ax. Give two specific examples of A and
explain the kinds of geometric operations thus involved respectively.
( 96)

(a)
AT A = [a1 a 2  a n ] [a1 a2  a n ]
T

a1T a1 2
T 2

a2 a2
=

[ a1 a 2  a n ] =


T
an 2
an

T
A A ()
ai 0 ()
(positive definite)()

a 0 0 1 0 0

(b) (scaling) 0 b 0 x- 0 cos sin
0 0 c 0 sin cos

1 0 0
yz-(reflection) 0 1 0
0 0 1


http://press.nctu.edu.tw

2 2 2
2. Assume A = 2 2 2 .
2 2 2

(a) Find the eigenvalues of A.


(b) Find the eigenvectors of A.
(c) Is A positive definite?
(d) Find An , where n is a positive integer.
( 97)


(a)

2 2 2
2 2 2 2 2 2
A I = 2 2 2 = (2 ) 2 +2
2 2 2 2 2 2
2 2 2
= (2 )( 2 4 ) + 4(2 ) = 2 ( 6)

= 0 06

1 1
(b) = 0 1 , 0 = 6
0 1

1
1
1

(c) = 0 06 A
(d)
1
1 1 1 0 1 1 1
n 1
A = S S = 1 0 1 0
n 1 0 1

0 1 1 6n 0 1 1

1 1 1 0 1 2 1 1 1 1
6n
A = 6 1 0 1 0 1 1 2 = 1 1 1

n n 1
3 3
0 1 1 1 1 1 1 1 1 1


http://press.nctu.edu.tw

0 1 1
3. Consider the matrix Q = 1 0 1 .
1 1 0

(a) Is this matrix positive definite, negative definite, or indefinite?


(b) Is this matrix positive definite, negative definite, or indefinite on the

subspace M = {x : x1 + x2 + x3 = 0} ?

(c) Consider the quadratic form

f ( x1 , x2 , x3 ) = x12 + x22 + 5 x32 + 2 x1 x2 2 x1 x3 + 4 x2 x3 . Find the values of the

parameter for which this quadratic form is positive definite.


( 97)

(a) Q 1 1 2
Q (indefinite)
(b) M x 0 M x1 + x2 + x3 = 0

0 1 1 x1 x1
x Qx = [ x1
T
x2 x3 ] 1 0 1 x2 = [ x1 x2 x3 ] x2
1 1 0 x3 x3
= ( x12 + x22 + x32 ) < 0

(negative definite)

1 1 x1
(c) f ( x1 , x2 , x3 ) = 1 2 x2
1 2 5 x3

(a)

1 =1> 0

1
= 1 2 > 0 1 < < 1
1

1 1
1 2 2 1 4
1 2 = 1 = 5 2 4 > 0 < < 0
2 5 1 5 1 2 5
1 2 5


http://press.nctu.edu.tw

4
< < 0
5

http://ccjou.twbbs.org/blog/?p=3736

4. True or false.
(a) Let S be the set of ordered pairs of real numbers. Define scalar

multiplication and addition on S by ( x1 , x2 ) = ( x1 , x2 ) and

( x1 , x2 ) + ( y1 , y2 ) = ( x1 + y1 , 0 ) , respectively. Then S is a vector space.

(b) Let R denote the set of all real number. If S is a closed and bounded interval
in R and contains 0, then S is not a subspace of R.
(c) Let A be an n n matrix. If det( A) = 0, then Ax = 0 must have a
nontrivial solution.
(d) Let A be an m n matrix. If B is a nonsingular n n matrix, then AB and
A have the same nullspace.
(e) Let A, B, S and T are n n matrices. Suppose that A = ST , where S is
nonsingular, and B = TS , then A is similar to B.

(f) Let A be an m n matrix of rank n, and let P = A ( AT A ) AT . If


1

b R ( A), the range of A, then Pb = 0.


(g) Suppose the n n matrix A is positive definite in the n dimensional real
space R n . Let B be an n m matrix with rank m, then BT AB is a
positive definite matrix in R m .

(h) Consider the vector space R n with inner product x, y = xT y. Suppose

N ( A) = {0} , where N ( A) denotes the nullspace of the n n matrix A.

Then AT Ax, x = 0 if and only if x = 0.

( 96)

(a) 8

( + )( x1 , x2 ) = ( x1 , x2 ) + ( x1 , x2 )


http://press.nctu.edu.tw

( + )( x1 , x2 ) = ( ( + ) x1 , ( + ) x2 )

( x1 , x2 ) + ( x1 , x2 ) = ( x1 , x2 ) + ( x1 , x2 ) = ( ( + ) x1 , 0 )

(b) S 0 S = {0}


(c) det( A) = 0 A rank( A) < n
dim N ( A) > 0 Ax = 0 x 0
(d) B AB A ABx = 0 Ax = 0

A = [1 2]

1 0 2
B= AB = [1 4] N ( A) = span
0 2 1

4
N ( AB) = span
1
(e) A B P A = PBP 1 A = ST
S AS = STS = SB S A = SBS 1

(f) b R ( A) b A P = A ( AT A ) AT
1

R m A Pb = b

(g) A x 0

xT Ax > 0 By 0 y T BT ABy = ( By ) A ( By ) > 0


T

m y 0 By 0 B n m
rank( B) = m B y 0
By 0 BT AB

(h) AT Ax, x = ( AT Ax ) x = xT AT Ax = ( Ax ) Ax = Ax
T T 2

N ( A) = {0} Ax = 0 Ax = 0 x = 0
2


http://ccjou.twbbs.org/blog/?p=4807


http://press.nctu.edu.tw

3 2 0
5. Let A = 2 3 0 .
0 0 3

(a) Find the characteristic polynomial of matrix A.


(b) Find the eigenvalues and corresponding orthonormal eigenvectors of A.
(c) Give the spectrum decomposition for A.
(d) Find the singular values of A.
(e) Find A3 + 3 A2 13 A 17 I .
( 96)

3 2 0
(a) A A I = 2 3 0
0 0 3

A 0
= A B
0 B

3 2
A I = 3 = (1 + )( 5 + )( 3 ) = 3 3 2 + 13 + 15
2 3

(b) (a) 1 3 5 A

1 0 1
1 1
1 0 1 A A = QQT
2 2
0 1 0

1 0 0 1 2 0 1 2

= 0 3 0 Q = 1 2 0 1 2
0 0 5 0 1 0

(c) (spectral decomposition) 1

1 q1
T

A = QQT = [q1 q 2 q 3 ] 2 qT = q qT + q q T + q qT
2 1 1 1 1 2 2 1 3 3

3 qT3

(b)


http://press.nctu.edu.tw

1 2 1 2 0 0 0 0 1 2 1 2 0
A = ( 1) 1 2 1 2 0 + 3 0 0 0 + ( 5 ) 1 2 1 2 0

0 0 0 0 0 1 0 0 0

(d) AT A (semi-definite) i

i = i (singular value) A (b)

A = QQT AT A = A2 = Q 2QT AT A 1925 A


135
(e) (b) A = QQT

( )
A3 + 3 A2 13 A 17 I = Q 3 + 3 2 13 17 I QT


(a)

A = S S 1

p( ) = ( 1 )( 2 )( n )

p( A) = ( 1I A)( n I A) = S ( 1 I ) S 1  S ( n I ) S 1

S 1 S p( A) = S ( 1I )( n I ) S 1

1 1 n 1

( 1I ) ( n I ) =


1 n n n
(1 1 ) (2 1 ) (n 1 )
=


(1 n ) (2 n ) (n n )
0
=


0

A p ( A) = 0 p( A)
A
Cayley-Hamilton

A3 + 3 A2 13 A 17 I = ( A3 + 3 A2 13 A 15 I ) 2 I = 0 2 I = 2 I


http://press.nctu.edu.tw

http://ccjou.twbbs.org/blog/?p=3105

6. A matrix A R nn is said to be idempotent if A2 = A. Suppose that A is an


idempotent matrix.
(a) Show that each eigenvalues of A is either 0 or 1.
(b) Show that A is diagonalizable.
(c) What is the minimal polynomial of A?
(d) If 0 is an eigenvalue of A with algebraic multiplicity k, k n, what is
rank( A) ?
( 96)


(a) A Ak k (k 1) A2 = A 2 =
= 0 1
(b) (1)
(2)
(3)
n

(1)(a)
(2)
(3)
(c)

(c)
(primary
decompodition)(
)(
)
(c)
m( ) = ( 3)( 2)2 2 2 3
2


http://press.nctu.edu.tw

() A

(3)

A2 = A
(c)
A
(c) A (minimal polynomial)


(1)
2 1 0 0
0 2 0 0
A =
0 0 1 1
0 0 2 4
p ( ) = ( 3)( 2)3 m( ) = ( 3)m ( 2)n
m n (2) ( m + n)
A m( A) = ( A 3I ) m ( A 2 I ) n = 0 A m ( A)

Cayley-Hamilton p( A) = ( A 3I )( A 2 I )3 = 0
f ( ) = ( 3)( 2)
g ( ) = ( 3)( 2) 2 h( ) = ( 3)( 2)3 A
f ( A) = ( A 3I )( A 2 I ) 0 g ( A) = ( A 3I )( A 2 I ) 2 = 0
m( ) = ( 3)( 2)2

(a) 0 1
p ( ) = ( ) k (1 ) n k k = 0 k = n k 0 k n

A2 = A A p = A ( I A ) = I A p > 1
p


k = 0 A 1 A A2 = A A1
A = I Cayley-Hamilton p ( A) = ( I A) n = I A = 0
m( ) = (1 )
k = n A 0 Cayley-Hamilton
p ( A) = An = A = 0 m( ) =
k 0 k n p ( A) = Ak ( I A)n k A( I A) = A A2 = 0
m( ) = (1 )


http://press.nctu.edu.tw

(b)

(d) m m
p < m

(algebraic multiplicity) m(geometric
multiplicity) p p 0 k
(b) A k
Ax = 0 k
A k rank( A) = n dim N ( A) = n k

http://ccjou.twbbs.org/blog/?p=3667

0 1
7. Find a singular value decomposition for the matrix M = 1 1 .
1 0

( 97)

(singular value decomposition) M = U V T



(1) (cross product)

0 1
0 1 1 2 1
M M =
T
1 1 =
1 1 0 1 0 1 2

(2) M T M M (singular value)

2 1
= ( 2 ) 1
2

1 2

1 1 1 1
1 = 1 1 = 1 v1 = 1 2 = 3 2 = 3 v 2 =
2 2 1

(3) V

1 0
1 2 1 2
V = [ v1 v2 ] = = 0 3
1 2 1 2 0 0


http://press.nctu.edu.tw

(4) U

0 1 1
1 1 1 1 1
u1 = Mv1 = 1 1 = 0
1 1 2 1 2
1 0 1

0 1 1
1 1 1 1 1
u2 = Mv 2 = 1 1 = 2
2 3 2 1 6
1 0 1

1
1
u3 = 1
3
1

1 1 1

2 6 3
2 1
U = [u1 u 2 u3 ] = 0
6 3
1 1 1
3
2 6
(3)(4) M = U V T

http://ccjou.twbbs.org/blog/?p=3326


http://press.nctu.edu.tw

You might also like